Review Questions and Case Studies

अब Quizwiz के साथ अपने होमवर्क और परीक्षाओं को एस करें!

what are the major goals and interventions to minimize fall risk factors?

-ID fall risk -Increase strength and flexibility -Implement balance and gait training -Compensate for sensory deficiencies -Functional training -ADs as needed -Safety educatiion -Modify environment

What are the goals of inpatient cardiac rehab in the acute stage?

-3-5 days -Early return to independence in ADL -Counteract effects of bed rest by reducing risk of thrombi -Maintain muscle tone -Reduce orthostatic hypotension -Maintain joint mobility -Reduce anxiety -Patient education -Promote risk factor reduction

what are two conditions that may warrant a PT recommending a motorized tilt-in-space wheelchair for a patient?

-Allows the seat and back to be tipped backward as a unit -Indicated for pts with significant extensor spasms that could eject the pt from the chair or for pts who cannot perform pressure reliefs independently

Compare and contrast payment mechanisms used in managed care

-Capitation: payment to primary care provider is based solely on the number of enrolled beneficiaries; their care is managed using these funds, if enrollees remain healthy provider profits are larger -Fee for service: providers are paid after services are provided to the patient -Case based rate: payment to providers is made based on the diagnosis, often payment is bundled by the day, the visit, or episode of care

What are the adverse side effects of cancer treatment that can impact a PT session?

-Chemotherapy: fatigue, GI sxs, bone marrow suppression, skin rashes, neuropathies, phlebitis, leukopenia increases risk of infection, thrombocytopenia increases risk of bleeding, anemia can decrease aerobic capacity -Long term corticosteroid use: immunosuppression, increases risk of DM, HTN< obesity, OP, easily bruised -Immunosuppression: fatigue, weight loss, flu-like sxs, nausea, anorexia, edema -Radiation: pain, fatigue, fibrosis, burns, delayed wound healing, edema, NS effects

What are the characteristics in terms of communication, gait, tone, balance, and respiratory function in a patient with late PD (Stage IV)?

-Communication: dysarthria, hypophonia, mask-like face, small writing -Gait: poverty of movement, festinating gait possible -Tone: cogwheel rigidity -Balance: impaired postural reactions, trunk rigidity, lack of rotation -Respiratory function: decreased chest expansion, decreased vital capacity

When managing edema secondary to lymphatic dysfunction, use of manual lymph drainage (MLD) emphasizes which directional principles?

-Emphasis on decongesting proximal segments first, then extremities -Flow within segments is distal to proximal

Which characteristics of US application affect the depth of penetration and thermal effects?

-Frequency in MHz determines depth of penetration (3 MHX in superficial, 1 MHz in deep) -Temporal characteristics are major in determining thermal characteristics -Pulse US produces less heat and is considered nonthermal -Continuous US produces thermal effects and increases tissue temp

Which cranial nerves may play a role in vision? What findings are normal or abnormal?

-II/Optic: visual acuity, visual fields, pupillary constriction -III/Oculomotor: pupillary size, extraocular movement -IV/Trochlear: extraocular movement -V/Trigeminal: corneal reflexes -VI/Abducens: extraocular movement -VII/Facial: ability to close eyes tightly -VII/Vestibulocochlear: nystagmus

What type of equipment and ambulatory aids might be needed to progress a morbidly obese pt from a sedentary bed-bound situation to independent ambulation?

-Heavy duty mechanical lift (Hoyer lift) to help transfer the pr from STS -BWS with an overhead harness to unload some body weight in a progressive manner -Heavy duty, extra wide walker

what are the most important components of the initial exam of a pt with dementia?

-Hx: onset and progression of sxs -Cognitive function -Impairments of communication and perception -Behavior changes -Seld care -Motor function -Environmental safety

What are the 3 possible interventions a PT may use in the management of stress incontinence?

-Kegel's strengthening exercises -Functional e stim -Biofeedback

What are the typical meds that may be prescribed for a pt diagnosed with GERD?

-Proton pump inhibitors (prilosec) -H2 blockers (Tagamet) -Antacids (tums)

What are some of the long term complications of DM that a PT needs to consider during examination and treatment?

-Retinopathy, renal disease, polyneuropathy, atherosclerosis, CVA< MI, PAD, joint stiffness, OP< GERD< liver disease, skin ulcers, amputations, etc.

Based on CPG For knee ligament sprain, which intervention has the strongest overall evidence for effectiveness

-Therapeutic exercise: NWB open chain and WB closed chain activities

following a THR, in the acute phase, which positions should be avoided in bed positioning and activities involving bed mobility

-wedge to prevent hip adduction -hip flexion < 90* -limit adduction and IR when moving in bed

A patient with bilateral lower extremity swelling complains of chest pain. When the PT auscultates the patient's heart, a S3 heart sound is heard. When is S3 heart sound heard and what condition does it LIKELY indicate? A. Abnormal sound heard in early diastole; congestive heart failure B. Abnormal sound heard in late diastole; congestive heart failure C. Abnormal sound heard in early diastole; myocardial infarction D. Abnormal sound heard in late diastole; hypertension

A. Abnormal sound heard in early diastole; congestive heart failure

A patient underwent surgical repair of a SLAP lesion 10 days ago. Which of the following interventions is LEAST appropriate at this time? A. Active isometric contraction of the biceps B. Passive humeral rotation with shoulder in scapular plane C. Pendulum exercises within the tolerated range D. Active isometric contraction of the triceps

A. Active isometric contraction of the biceps

What risk factors contribute to the development of coronary artery disease?

Age, gender, family history, cigarette smoking, sedentary lifestyle, obesity BMI>30, HTN, dyslipidemia, prediabetes, increased plasma glucose, DM

A physical therapist is working with a 67-year-old male retired police officer. The patient displays rigidity, slowed movements, involuntary oscillatory movements, and instability that has led to a recent uptick in falls. Based on the patient's presentation which of the following neurological disorders could the patient MOST likely have? A. Parkinson's Disease B. Guillain-Barré syndrome C. Multiple Sclerosis D. Amyotrophic lateral sclerosis

A. Parkinson's Disease

A 40-year-old male patient presents with midback pain, stiffness localized to the pelvis and hips. The pain is worse in the morning lasting for 1 hour and increased at SI joints. Patient's lab report is positive for HLA B27 antigen in blood. What is the MOST appropriate diagnosis? A. Ankylosing spondylitis B. Osteogenesis imperfecta C. Scheuermann's disease D. Spondyloepiphyseal dysplasia

A. Ankylosing spondylitis

When examining DTRs, what is the score for an obligatory and sustained response?

4+ 0: absent 1+:tone change no movement 2+: visible movement 3+: exaggerated full movement 4+: obligatory and sustained >30 sec

A 56-year-old male has hyperlipidemia, CHF, HTN. During the evaluation, the patient provided list of medications with one of them being Ramipril. The therapist is educating on potential side effects from this medication as the following side effects MOST likely correlate with this drug: A. Causes hyperkalemia B. Causes hypokalemia C. Increases blood pressure D. Regulates heart rate

A. Causes hyperkalemia

A PT commands a patient to pick a pen amongst straws, but he is unable to complete the task due to inability to find pen. Which of the following is the MOST APPROPRIATE diagnosis to document this deficit? A. Form discrimination B. Figure-ground discrimination C. Position in space impairment D. Ideational apraxia

A. Form discrimination

A patient presents to a clinic with Subacromial impingement syndrome. The PT chooses to use ultrasound for deep heating of the rotator cuff muscles. Which set of US parameters will be MOST effective at increasing the temperature of tissue 4.0 centimeters deep? A. 1.0 MHz, 100% duty cycle with 5cm sound head B. 3.0 MHz, 100% duty cycle with 2cm sound head C. 1.0 MHz, 50% duty cycle with 2cm sound head D. 3.0 MHz, 50% duty cycle with 5cm sound head

A. 1.0 MHz, 100% duty cycle with 5cm sound head

A 58-year-old female patient with uncomplicated MI (Myocardial Infarction) has been discharged from the acute care. Which of following activities would be MOST appropriate during early phase 2 rehabilitation? A. 30 minutes walking at 3 mph with no incline. B. 15 minutes of elastic band training with yellow elastic bands. C. 15 minutes upper limb workout with 2-pound hand weights. D. 30 minutes walking at 5 mph with 5% incline.

A. 30 minutes walking at 3 mph with no incline.

A 32-year-old female patient arrived at a clinic with the following skin disorder. Which of the following is the MOST appropriately describe these findings? A. Herpes simplex virus type 1 B. Herpes zoster C. Acne vulgaris D. Herpes simplex virus type 2

A. Herpes simplex virus type 1

A PT notices that a patient is experiencing early toe-off during terminal stance in gait. Which of the following identifies a likely cause, AND an appropriate intervention to address that cause? A. Hip flexion contracture, prolonged stretch B. Hip adductor weakness, progressive strengthening C. Gastrocnemius weakness, ultrasound D. Great toe flexion weakness, progressive strengthening

A. Hip flexion contracture, prolonged stretch

During the examination, the patient reports increased pain while standing. Pain is alleviated with sitting. Which special test will be positive to confirm the diagnosis shown in the picture ? A. Stork standing test B. Gillet Test C. Van Gelderen bicycle test D. Quadrant Test

A. Stork standing test

In the developing infant, what are the differences of age of onset and response between the ATNR and STNR?

ATNR: present at birth, rotation of the head results in flexion of skull side limbs and extension of face side limbs STNR: appears between 4-6 mos of age, cervical flexion results in flexion of arms and extension of legs, cervical extension results in extension of arms and flexion of legs,

A PT is administering the six-minute walk test to assess the aerobic capacity of a patient with a four-year history of chronic obstructive pulmonary disease. Which of the following statement regarding the test is MOST accurate? A. The PT must walk with the patient to observe SpO2 on the pulse oximeter. B. "Warm-up" or practice test should not be performed immediately before the test. C. The patient is allowed to take rest/ breaks with the timer paused D. The results of this test help to determine the cause of dyspnea.

B. "Warm-up" or practice test should not be performed immediately before the test. Need to wait an hour between

The PT is treating a 43-year-old male who sustained a left hip fracture. The patient is having difficulty gaining hip flexion ROM. How much minimum hip flexion should the PT try to obtain so that the patient can ambulate with a normalized gait pattern? A. 15 degrees B. 30 degrees C. 40 degrees D. 60 degrees

B. 30 degrees

A 46-year-old female patient presents to the PT clinic with chief complaints of swelling in the wrist, elbow and MCP joints. The symptoms have been present since the last six weeks. The patient also mentioned that she had an episode of fever a few weeks ago and feels tired all the time. Which of the following symptoms are LEAST likely to be expected along with this presentation? A. At least one hour of stiffness in the morning before improvement in symptoms B. Asymmetrical involvement of bilateral metacarpophalangeal joints and proximal interphalangeal joints C. Positive serum rheumatoid factor D. Bony decalcification adjacent to affected joints

B. Asymmetrical involvement of bilateral metacarpophalangeal joints and proximal interphalangeal joints

A PT is teaching a patient with Stage 5 Parkinson disease on how to wheelchair transfer to the commode. The patient has very low endurance. What motor learning strategy would work best for this patient to learn this task? A. Random order and distributed practice B. Blocked and distributed practice C. Random and massed practice D. Blocked and massed practice

B. Blocked and distributed practice

A physical therapist applied saline solution and sugar solution in effort to assess sensory to posterior 1/3 of tongue. Which of the following cranial nerves would MOST LIKELY be tested? A. CN VII B. CN IX C. CN V D. CN VI

B. CN IX

A patient has reddish brown papules and plaques around the mouth. The area is warm and tender to the touch. Which of the following conditions does the patientMOSTlikely have? A. Cryoglobulinemia B. Cutaneous sarcoidosis C. Emphysema D. Vasculitis

B. Cutaneous sarcoidosis

A PT is using aquatic therapy to treat a 29-year-old patient who had a recent ACL repair. The patient is immersed to the level of sternoclavicular notch. Which of the following is the MOST expected physiological response of aquatic therapy? A. Decreased cardiac output B. Decreased systolic blood pressure C. Increased heart rate D. Increased V02max

B. Decreased systolic blood pressure

A PT examines a 59-year-old patient with sub-acute stroke. She cannot drink from the bottle on command and automatically. What sort of perceptual disorder does this patient have? A. Ideomotor apraxia B. Ideational apraxia C. Form discrimination D. Topographical disorientation

B. Ideational apraxia

The PT is observing the gait of a patient and notes that the patient slaps the forefoot on the ground during the early stance phase of the gait cycle. Which of the following impairments of the orthotic could potential contribute to this presentation? A. Excessive dorsiflexion assist B. Inadequate dorsiflexion assist C. Inadequate knee lock D. Excessive plantarflexion stop

B. Inadequate dorsiflexion assist

A PT is examining a 10-year-old patient diagnosed as level 3 on GMFCS. According to the gross motor classification, what is the MOST likely ambulation status of this patient? A. Patient will walk without restrictions but will have limitations in more advanced gross motor skills. B. Patient will walk with assistive device with limitations in walking outdoors and in the community. C. Patient self mobility will be severely limited, even with the use of assistive technology. D. Patient will walk without assistive device with limitations in walking outdoors and in the community.

B. Patient will walk with assistive device with limitations in walking outdoors and in the community.

A 36-year-old female is experiencing tenderness in her cervical spine at the level of C5-C6 vertebrae. The physician diagnoses reduced space between the vertebrae. What will be the MOST appropriate PT intervention? A. Provide an ice pack to relieve pain B. Perform cervical mobilization by moving the C5 vertebrae anteriorly C. Perform cervical mobilization by moving the C6 vertebrae anteriorly D. Provide ultrasound therapy to relieve pain

B. Perform cervical mobilization by moving the C5 vertebrae anteriorly

Two PT students are reviewing the literature for the effects of locomotor training to improve balance and walking function in people with stroke. According to levels of evidence, which studies provide the BEST evidence supporting use of locomotor training? A. Clinical case report B. Randomized controlled trials C. Clinical case series D. Cross sectional studies

B. Randomized controlled trials

A PT is auscultating by listening to various sounds of the heart. The therapist noticed closure of the aortic and pulmonary valves. Which of the following heart sounds correlates to the therapist description? A. S3 B. S4 C. S2 D. S1

C. S2

A 48-year-old male patient reports of shortness of breath and complains of fatigue. During the examination, the PT examines patient's heart sounds before starting an exercise program. Which valve is being auscultated in the picture? A. Tricuspid valve B. Pulmonary valve C. Mitral Valve D. Aortic Valve

B. Pulmonary valve

A 30-year-old patient is unable to control descent when moving from standing to sitting position. Which of the following interventions would be MOST appropriate to address this problem? A. Hamstring strengthening with concentric contractions B. Quadriceps strengthening with eccentric contractions C. Quadriceps strengthening with concentric contractions D. Gluteus medius strengthening with eccentric contractions

B. Quadriceps strengthening with eccentric contractions

A patient develops a Stage 2 pressure injury over the sacrum and is referred to physical therapy for wound care. Which of the following is the MOST appropriate initial application to clean the wound? A. Povidone-iodine solution B. Sterile normal saline C. Zinc oxide cream D. Nitrofurazone solution

B. Sterile normal saline

Two friends were admitted to a local hospital with medical diagnosis of head trauma after a skiing accident. They were recently transferred to skilled nursing facility. Upon examination, one of them demonstrates goal directed behavior relying on external input. This patient is MOST likely classified as: A.Level V B.Level VI C.Level IV D.Level III

B.Level VI

A study is looking at effects of different exercise options for a patient with knee OA. The study compared pool exercise, treadmill walking, and ankle weights on knee pain in 3 equal groups of 30 adults each. An appropriate statistical test to assess the effectiveness of treatment in three groups is: A. K Wallis test B. Two paired t-tests C. ANOVA D. Chi square

C. ANOVA

Two PTs perform a test on a patient using the Berg Balance test. The patient visit is Monday (M) and Wednesday (W). One PT reports scores of 29 and 30 on M & W; other PT scored 36 and 37 (M&W). This is indicative of a problem in: A. Test-retest reliability B. Intra-rater reliability C. Inter-rater reliability D. Split-half reliability

C. Inter-rater reliability

A 38-year female patient presents to an outpatient clinic with complains of knee pain. The PT suspects a ligament involvement. Which of the following is MOST appropriate diagnostic test to confirm these findings? A. Anterior Drawer test B. Radiography (X-Ray) C. Magnetic resonance imaging (MRI) D. Computed tomography (CT) scan

C. Magnetic resonance imaging (MRI)

A male has been diagnosed with as complete C5 spinal cord injury. He used a power chair to commute from the parking lot to the PT clinic. While performing a transfer, the MOST important safety consideration is to A. Make sure he is using his shoulder extensors to support his upper body B. Make sure his legs are supported by PT aide C. Make sure his power chair is turned off D. Make sure to use the sliding board

C. Make sure his power chair is turned off

A review of the patient's medical chart shows BP of 168/90 mm Hg, triglyceride level of 160 mg/dL, and a fasting blood glucose level of 115 mg/dL. Patients BMI is 40 kg/m2and his waistline is 54-inch. These findings are suggestive of: A. Chronic heart disease B. Type 2 diabetes C. Metabolic syndrome D. Stage I hypertension

C. Metabolic syndrome

A researcher is collecting data on 100 tennis players with potential rotator cuff injuries. Following values are provided True Positive-80% -True Negative-75% False Positive-25% -False Negative-20% What is the sensitivity and specificity of the test? A. Sensitivity is 20% and Specificity is 25% B. Sensitivity is 75% and Specificity is 80% C. Sensitivity is 80% and Specificity is 75% D. Sensitivity is 60% and Specificity is 40%

C. Sensitivity is 80% and Specificity is 75%

A PT is assessing a patient's lymph nodes 6 months postchemotherapy treatment. When assessing the lymph nodes, which presentations LEAST likely require referral to physician? A. Hard and immobile lymph nodes less than 1cm in diameter B. Rubbery and firm lymph nodes more than 1 cm in diameter C. Soft and non tender lymph nodes less than 1 cm in diameter D. Palpable and tender lymph nodes more than 1 cm in diameter

C. Soft and non tender lymph nodes less than 1 cm in diameter

The displayed graph shows trunk lean in obese patients. According to this graph, which of the following statements BEST describes the relationship between trunk lean and waist circumference? A. There is strong positive relationship between the two variables. B. There is weak negative relationship between the two variables. C. There is a strong negative relationship between the two variables. D. There is weak positive relationship between the two variables.

C. There is a strong negative relationship between the two variables.

Which of the following cranial nerves is MOST likely to be affected in a patient suspected to have a lesion in the pons? A. Oculomotor B. Optic C. Trigeminal D. Trochlear

C. Trigeminal

A 45-year-old obese male (BMI 33kg/m2) with type 2 DM is working out on the treadmill in a hospital setting. While exercising, the patient suddenly develops light-headedness, dizziness, and instability. The MOST appropriate action is: A. Stop the treadmill and call the primary care physician B. Keep the treadmill moving and have the patient drink orange juice C. Stop the treadmill and have the hospital nurse check blood glucose levels D. Slow the treadmill speed and have the patient drink cold water

C. Stop the treadmill and have the hospital nurse check blood glucose levels

A 40-year-old male patient reports of shortness of breath and swelling in LE. During the baseline examination, the PT examines patient's heart sounds before starting an exercise program. Which valve is being auscultated in the picture shown below? A.Mitral valve B.Pulmonary valve C.Tricuspid valve D.Aortic valve

C.Tricuspid valve

When auscultating the lungs, what are the main characteristics of crackles, wheezes, vesicular, and bronchial breath sounds?

Crackles: crackling sounds heard during inspiration that indicate pathology (pulmonary edema, fibrosis, atelectasis) Wheezes: musical sounds heard during expiration with COPD Vesicular: normal, soft rustling sounds heard throughout inspiration and beginning of expiration Bronchial: normal, hollow echoing sounds heard over the R mainstem bronchus during inspiration and expiration

Which of the following instruments is MOST appropriate for measuring patients' quality of life? A. Dynamic Gait Index (DGI) B. Oswestry low back pain disability index (ODI) C. Timed up and go test(TUG) D. Medical Outcomes Study 36-item short form (SF-36)

D. Medical Outcomes Study 36-item short form (SF-36)

PT assesses patient muscle tone, muscle coordination and muscle strength with scoring 2 on the Modified Ashworth Scale. Which of the following is the BEST interpretation based on this score? A. More marked increase in muscle tone through most of ROM however affected parts difficulty to move B. More marked increase in muscle tone through part of ROM however affected parts easily moved C. Considerable increase in muscle tone as it difficult to perform passive movements D. More marked increase in muscle tone through most of ROM however affected parts easily moved

D. More marked increase in muscle tone through most of ROM however affected parts easily moved

A 58-year-old male is performing a Bruce protocol in your clinic with ECG leads attached. During the protocol, the PT sees the picture shown below. What is the MOST likely diagnosis and intervention? A. Myocardial ischemia that has an elevation of the ST segment greater than 1 mm and the PT should stop the protocol B. Myocardial infarction that has an elevation of the ST segment less than 1 mm and the PT should stop the protocol and call 911 C. Myocardial ischemia that has an elevation of the ST segment less than 1 mm and the PT should stop the protocol D. Myocardial infarction that has an elevation of the ST segment greater than 1 mm and the PT should stop the protocol and call 911

D. Myocardial infarction that has an elevation of the ST segment greater than 1 mm and the PT should stop the protocol and call 911

A researcher is collecting ROM data on volleyball players during shoulder abduction motion. The results of the tests are BEST categorized as which of the following types of data? A. Interval data B. Ordinal data C. Nominal data D. Ratio data

D. Ratio data

A patient presents to the clinic with chief concerns of dizziness since the last 8 months. On assessment the patient has an ataxic gait and tests positive for dysdiadochokinesia. Head thrust test and positional testing had negative findings. Which of the following is the MOST appropriate intervention for this patient? A. Balance exercises B. VOR x 1 progressing to VOR x 2 in sitting C. VOR x 1 progressing to VOR x 2 in standing D. Refer the patient to a physician

D. Refer the patient to a physician

While evaluating the gait cycle of a 28-year-old female patient, the PT observes right pelvic hike during the swing phase of the right gait cycle. Which of the following condition is LEAST likely to cause the problem? A. Reduced right hip flexion B. Inadequate right knee flexion C. Lack of right ankle dorsiflexion D. Right ankle plantar flexor weakness

D. Right ankle plantar flexor weakness

A patient with a history of stab wound disrupting the left side of spinal cord is being evaluated by the therapist. Which of the following is the MOST LIKELY presentation for this patient? A. Symptoms of damage to corticospinal tract and spinothalamic tract seen on the R side of the body and symptoms of damage to the dorsal column medial lemniscus seen on the L side of the body B. Symptoms of damage to corticospinal tract and dorsal column medial lemniscus seen on the R side of the body and symptoms of damage to the spinothalamic tract seen on the L side of the body C. Symptoms of damage to corticospinal tract and spinothalamic tract seen on the L side of the body and symptoms of damage to the dorsal column medial lemniscus seen on the R side of the body D. Symptoms of damage to corticospinal tract and dorsal column medial lemniscus seen on the L side of the body and symptoms of damage to the spinothalamic tract seen on the R side of the body

D. Symptoms of damage to corticospinal tract and dorsal column medial lemniscus seen on the L side of the body and symptoms of damage to the spinothalamic tract seen on the R side of the body

A 44-year-old male patient is being evaluated by a PT. The patient reports referred pain in the left shoulder with diagnosis of a positive Kehr's sign. Which of the following is NOT a potential cause of a positive Kehr's sign and left shoulder pain? A. Recent laparoscopy B. Intra-abdominal bleeding C. Rupture of the spleen D. Trauma to head of pancreas

D. Trauma to head of pancreas

A patient with normal sensation and reflexes presents to the clinic with decrease in muscle tone and asthenia. Which of the following other signs and symptoms are MOST likely expected to be seen during the patient assessment? A. Bradykinesia B. Synergy C. Uniform resistance to passive range of motion D. Truncal ataxia

D. Truncal ataxia

A PT collected data on THA patients and plotted the relationship of trunk flexion with waist circumference. The regression plot showed a R square value of 0.64 (See picture). What is the best way to describe the slope? A.Weak positive slope B.Weak negative slope C.Strong positive slope D.Strong negative slope

D.Strong negative slope

which electrical stimulation characteristics are appropriate for wound healing?

HVPC and low-intensity low-volt continuous direct current

What is the common pattern of fatigue in many pts with MS?

High energy in early morning followed by early afternoon fatigue and exhaustion, and then some recovery by early evening Schedule these pts for morning intervention or consider this pattern for job or ADL tasks

Differentiate between hypothyroidism and hyperthyroidism in terms of expected symptoms?

Hypo: weight gain, lethargy, low BP, constipation, intolerance to cold, dry skin, appearance of goiter Hyper: nervousness, hyperreflexia, tremor, hunger, weight loss, fatigue, heat intolerance, tachycardia, diarrhea

What is the difference between independent and dependent variables?

Independent: a variable that stands alone and isn't changed by the other variables you are trying to measure; the variable being manipulated Dependent: the variable that is being studied and measured; it changes when the independent variable is manipulated

From highest to lowest level of rigor, what is the hierarchy of evaluating and grading levels of evidence?

Level 1/Grade A: systematic review including meta analysis, individual RCTs Level 2/Grade B: cohort studies Level 3/Grade B: case control studies, retrospective studies Level 4/Grade C: case series or poor quality cohort and case control studies; descriptive studies Level 5/Grade D: expert opinion or observations

What are the contraindications for use of e stim?

Near demand-type pacemakers, unstable arrhythmias, epilepsy, seizure disorder, active bleeding, near thrombophlebitis, superficial metal implants, over or near the carotid sinus, thoracic region, phrenic nerve, urinary bladder, low back during pregnancy, pharyngeal region

What are the PAO2 and FiO2 normal values in room air?

PAO2: 95-100 mmHg FiO2: 21%/0.21

A PT should be on the alert for which possible signs and symptoms of physical, emotional, and sexual child abuse?

Physical: bruises, burns, welts, injury marks from hand or belt, medical or dental issues untreated, child avoids touch or contact, seems fearful Emotional: depression, low self esteem, constant worrying about wrongdoing, behavioral extremes, disinterested in others Sexual: highly sexualized behavior, avoids certain people, bruising/bleeding/pain, pregnancy or STDs in those <14 years old

What is the significance of sensitivity and specificity regarding a clinical test

Sensitivity: a test's ability to correctly identify the proportion of all the patients that have the disease: a true positive This helps rule out a disease when the test is negative and the condition is not present: SnNOUT (Sensitive Negative OUT) Specificity: a test's ability to correctly identify the proportion of all the patients that do not have the disease: a true negative This helps rule in a disease when the test is positive and the condition is present: SpPIN (Specific Positive IN)

Differentiate between a superficial partial thickness and a full thickness burn

Superficial partial thickness: epidermis and upper dermis damage, bright pink or red, blisters, moderate edema, painful Full thickness burn: complete destruction of epidermis, dermis, and subcutaneous tissue, white/gray/charred appearance, poor circulation, dry, leathery surface, little pain since nerve endings are destroyed

What is the main principle being employed when using a PNF-facilitated stretching technique of contract-relax?

The muscle to be stretched relaxes as a result of autogenic inhibition possible from the GTO firing The muscle can be further relaxed through the effects of reciprocal inhibition if active contraction is performed

What is the significance of validity and reliability regarding a clinical test? What are the possible threats to each?

Validity: the degree to which a test actually measures what it's supposed to measure Threats to: sampling bias, lack of controls over the subjects, inaccurate measuring instrument, experimenter bias, variables in treatment administration, placebo effect, Hawthorne effect Reliability: the degree to which independent measurements of a given behavior are consistent, accurate, dependable, without variation Threats to: deal with inter/intra rater factors, errors of measurement, and systemic or environmental factors

Therapeutic intervention Case: Pt is a 35 year old male referred to PT with a dx of R biceps brachii grade II muscle strain. He sustained an injury paying softball 3 weeks ago and was initially placed in a sling for suspected rotator cuff tear. He now has full ROM of RUE but weakness of biceps brachii d/t inactivity (MMT 3+/5). No current pain with ADLs. No PMHx involving RUE. Denies neck pain or any neurologic symptoms. What is the typical healing timeline of a grade II muscle strain? a. 0-4 wks b. 3-12 wks c. 4 wks-6 mos d. 6-12 mos

b. 3-12 wks

When considering the concave-convex rule, to which joints in the spine does the concave rule apply

every joint below the 2nd C vertebra, only the OA joint is convex on concave

Which motion at the GH joint has the greatest limitation if the dx is adhesive capsulitis?

external rotation (capsular pattern)

When performing an exam of the skeletal system in a full term neonate, what possible abnormal bony condition should be part of the screening process?

fractured clavicle, hip dysplasia, spinal curvature, spina bifida occulta, talipes equinovarus

what are the physiological changes that may occur in the visual system in older adults?

general decline in visual acuity, presbyopia, decreased ability to adapt to light and dark, diminished oculomotor responses, cataracts, glaucoma

what are the major side effects of the use of pain meds in the geriatric population that can be a concern for PTs?

increased fall risk, disorientation, sedation

Which special test when applied to the ankle/foot is best used to identify ligamentous instability of the calcaneofibular ligament

talar tilt

What are the most common errors associated with training a pt to improve postural stability?

-Inadequate stretching of tight pelvic and hip muscles -Inadequate core muscle control -Starting at too high a functional level or progressing too rapidly -Exercising past the point of fatigue

what risk factors are associated with development of osteoporosis?

Hormonal deficiency associated with menopause, nutritional deficiency of calcium, excessive alcohol and caffeine consumption, decreased physical activity, hyperthyroidism, DM, celiac dx, corticosteroids, thyroid hormone, family hx

What are the initial PT goals and interventions upon receiving a referral for a patient recently dx with a unilateral vestibular disorder?

Implement safety measures first Teach sensory substitution and compensatory strategies and provide an ambulatory aid as indicated Habituation training, eye and head exercises, postural stability activities

When performing a physical exam of pressure injury, what elements should be part of the exam

Location of wound, assess length, width, depth, girth, examine for tunneling, determine type, amount, color, odor of exudate, presence of necrotic or granulation tissue, determine wound temp

MSK case study 2: pt is 68 year old female, referred to PT with cc of R posterior thigh/knee pain for past 6 weeks, onset after long period of sitting, knee stiffness on waking in morning that lasts <30 min, pain worsens with inactivity and improves with activity, Hx of L knee replacement 2 years ago, DVT ruled out, denies any neuro symptoms or L What elements of the pt hx make a dx of knee OA most likely? a. knee stiffness upon first waking up that lasts < 30 min b. knee stiffness upon first waking up that lasts > 60 min c. report of bilateral pain affecting multiple joints d. joint swelling, warmth, stiffness

a. knee stiffness upon first waking up that lasts < 30 min

Peds Case: Pt is a 12 month old male referred to PT for EI services and dx of down syndrome. He was born at full term via C-section, has a ventricular septal defect MD hopes will close, receiving speech therapy. Child is in foster care, caregiver has many questions, fine motor skills are age appropriate. He can belly crawl, sit independently, get into quadruped, and is starting to pull to stand. Prefers to sit in W and sleep prone with hips abd, ER, and flex. Transitions into sitting from prone by pushing up on hands and max abd hips. What is the prognosis for children with DS in terms of independent ambulation? a. Most will walk by 5 but will typically require an AD b. All children with DS will walk, likely by 2 c. It is highly variable, some will not walk d. Children with DS typically walk on time

b. All children with DS will walk, likely by 2

Neuro Case 1: 63 year old male. Pt referred to IP PT secondary to R ischemic stroke with L hemiparesis, admitted 3 days ago with intense HA and LUE>LE weakness exhibits L neglect. PMHx HTN, B knee OA, R knee arthroscopy. He is a college basketball coach, married and lives in a 2-story home. Which of the following arteries is occluded and most closely associated with the patient's presenting symptoms? a. Anterior cerebral artery b. Middle cerebral artery c. Posterior cerebral artery d. Basilar artery

b. Middle cerebral artery

Other Systems Case: Pt is a 64 year old male referred to OP PT with LBP x 5 years with increased sxs over the past year, no known MOI. Rates pain as a constant 4/10 with no consistent aggravating or easing factors, denies numbness, tingling, or weakness in LEs, denies BB incontinence but reports frequent urination, nocturia, and difficulty starting urination. PMHx: well controll HTN and high cholesterol. He is a married mechanic that does not smoke and rarely drinks. The pt is diagnosed with prostate cancer and undergoes surgery and radiation. The pt is referred for treatment secondary to significant pelvic floor dysfunction. What is the BEST option for initially starting pelvic floor exercises? a. Perform Kegel exercises while voiding b. Perform Kegel exercises in supine with knees bent c. Perform Kegel exercises in sitting d. Perform Kegel exercises everytime they move from sit to stand

b. Perform Kegel exercises in supine with knees bent

MSK case study 1: pt is 49 year old female, referred to OP PT with cc of R sided neck pain, insidious onset 3 days ago, pain and numbness extending to R posterior forearm and dorsum of hand and occasionally to 3rd digit, arm weakness, hx of HTN and occasional HAs with neck pain, school teacher, BMI 34, occasional alcohol consumption What elements from the patient interview make a dx of C radic most likely? a. associated referred pain to RUE b. acute onset of unilateral neck pain c. dermatomal paresthesia or numbness and myotomal muscle weakness d. Hx of HAs associated with neck movement

c. dermatomal paresthesia or numbness and myotomal muscle weakness

How would you instruct your patient regarding possible circumstances that could elicit angina pectoris?

-Increased demands on the heart could elicit angina pectoris: physical exertion, emotional stress, smoking, temperature extremes (cold), overeating

What are the implications of the FITT equation in terms of strategies to develop CV endurance?

-Intensity is the primary way to increased CV endurance -At least 2 days of exercise a week or more is needed to increase endurance -Duration of exercise depends on initial fitness level -Goal would be 20-30 mins, 3-5 days/wk -Initial time would be significantly altered for deconditioned or obese pts -Type of exercise should involve large muscle groups

What is defined as protected health information and what actions must be taken to protect it?

-PHI: individually ID health information, held or maintained by a covered entity or its business associates acting for the covered entity, that is transmitted or maintained in any form or medium -Locking cabinets or doors, FAX cover sheets, maintaining password protection on computers, conscious discussing pt information

What are the pressure tolerant areas in the typical transtibial residual limb that are suitable for a total contact socket?

-Patellar tendon -Medial tibial plateau -Tibial and fibular shafts -Distal end of the residual limb (can be problematic if open wound or sensitivity issues)

What are the important clinical changes affecting prognosis for a pt with diffuse systemic sclerosis

-Poorer prognosis -Integ changes more rapid and widespread -Earlier involvement of visceral organs (kidneys, heart, lungs) -Earlier mortality -Monitoring for acute HTN is essential

What are the expected ST segments changes following an acute myocardial infarction and with impaired coronary perfusion?

-Post MI: ST segment will be elevated -Impaired coronary perfusion: ST segment becomes depressed and can be upsloping, horizonal, or downsloping

How does rearfoot posting in a foot orthosis control for valgus or varus?

-Rearfoot posting acts primarily on the subtalar joint from initial contact to loading response -Medial wedge/varus post: limits or controls calcaneal eversion and IR of the tibia -Lateral wedge/valgus post: control an excessively inverted or supinated calcaneus and subtalar joint

Following a CVA, what are the major considerations when examining the patient for perceptual deficits?

-Testing for visual field deficits such as homonymous or bitemporal hemianopsia -Examine for body scheme or image such as unilateral neglect or somatognosia -Examine for spatial relations such as fatigue-ground, depth perceptions, visual disorientation, etc. -Examine for agnosia such as inability to recognize familiar objects with an impaired sensory modality -Examine for apraxia

Following an MVA, a pt with a complete SCI at the C7 level has been admitted to a rehab facility after a lengthy stay at an acute care hospital. What are the components of the PT's initial examination?

-Vitals -Respiratory function -Skin condition -Muscle tone -DTRs -Sensation -Muscle strength -Functional status -Wheelchair skills -Standardized tests: FIM

When exercising a patient with diabetes and coronary artery disease who is taking beta blockers, what is the best measure to monitor exercise performance?

Borg's RPE scale: increases linearly with increasing exercise intensity and correlates closely with HR and work rate HR is not reliable d/t beta blockers bunting HR response to increasing intensity

What are the precautions when using postural drainage in the Trendelenberg position?

CHF, significant HTN, PE, increased ICP, aneurysm, severe SOB, obesity, ascites, pregnancy, hiatal hernia

What are the 3 health conditions that may benefit from spinal mechanical traction?

Cervical radiculopathy Chronic neck pain with mobility deficits Lumbar radiculopathy that does not have directional preference

what are the increased and decreased physiological responses to the application of heat and cold?

Heat: Increased: blood flow, capillary permeability, elasticity, metabolism, edema Decreased: joint stiffness, muscle strength, muscle spasm, pain Cold: Increased: joint stiffness, pain threshold, blood viscosity Decreased: blood flow, capillary permeability, elasticity, metabolism, muscle spasm, muscle strength, spasticity

Differentiate between the viral infections herpes simplex and herpes zoster in terms of expected symptoms

Herpes simplex: preceded by itching and soreness followed by vesicular eruptions (cold sore) Herpes zoster: shingles, result of reactivation of varicella-zoster virus, results in pain, tingling, vesicle formation along dermatomes of spinal or cranial nerves, may last a long time

What are the typical LE contractions in a child with spastic CP?

Hip flexors, adductors, IR Knee flexors Ankle PF

Which categories of wound dressings can be used for exudative wounds

Hydrocolloids, hydrogels, foams, alginates, gauze Not used: transparent films

During strength training, what is the influence of the Valsalva maneuver regarding intrathoracic pressure, HR, venous pressure, and cardiac work?

Increased intrathoracic pressure Decreased HR Increased venous pressure Increased cardiac work

what are the muscle activation patterns during heel strike (initial contact) and heel off (terminal stance) for the quads, pretibial muscles, and plantarflexors?

Initial contact: quads are active for shock absorption and to control knee flexion, pretibial muscles control PF by decelerating the foot Terminal stance: peak activity of the PFs to generate forward propulsion

As a PT practicing in a healthcare facility, what are possible internal and external sources of disasters that might trigger the initiation of an emergency action plan (EAP)?

Internal sources: power, utility, communication or equipment failues, fire, gas leak, hazardous material, explosions, workplace violence External sources: terrorism, hate crimes, civil disturbances, mass casualty incidents, natural disasters

Identify 3 risk factors in each of the following categories that increase the likelihood of falls in older adults: medications, personal risk factors, chronic diseases

Medications: -Psychotropic drugs -Sedatives/hypnotics for antianxiety -Antipsychotics increase fall risk d/t syncope, sedation, slowed reflexes, loss of balance, and impaired psychomotor function -Antidepressants can cause hyponatremia: independent risk factor for falls -Anticonvulsants -Opioid analgesics can increase fall risk -Medications that affect BP/antiHTN can cause or worsen drop in BP/postural hypotension -Meds that lower blood sugar in older adults with DM resulting in hypoglycemia -Diuretics can cause orthostatic hypotension, risk is increased with dehydration Personal risk factors: -Visual impairments -Changes in postural BP -Impairments in balance -Impairments in gait -Foot pain or poor footwear -Sensory changes: decreased proprioception -Impairments in cognition -Impairments in muscle strength -Impairments in flexibility and ROM (LEs, feet) -History of inactivity or risky behaviors -Alcohol or drug use -Fear of falling Chronic disease: -Chronic health conditions: PD, arthritis, stroke

what are the major differences in dx characteristics and pattern of joint dysfunction between OA and RA

OA: -first manifested by changes in joint cartilage with eventual erosion to subchondral bone -all joints not equally affected -primary sites: DIPs, PIPs, CMC of thumb, C and L spine, hips, knees, and MTPs RA: -autoimmune disease that primarily affects the synovium, which proliferates, dissolves collagen, and extends over the joint cartilage -persistent inflammation and systemic complaints such as morning stiffness, fever, loss of appetite -joints affected in bilateral symmetrical pattern -all joints including TMJ and spine can be involved

What are the expected physical, imaging, PFT, and lab findings associated with moderate to severe asthma?

Physical: productive cough, dyspnea, decreased breath sounds, wheezes, crackles, tachypnea, tachycardia, increased accessory muscle use, anxiety Imaging: hyperlucency and flattened diaphragm PFT: dereased FEV1/FVC, decreased FEV1, lower flow rates Labs: ABG hypoxemia and hypercapnea with severe dx

Differentiate between venous and arterial ulcers in terms of expected clinical presentation

Venous: irregular shape and dark appearance, shallow, at medial malleolus, pulses present, little pain, fair amount of exudate Arterial: smooth edges, deep, often on toes, laterll malleolus, and shin, pulses often absent, painful, no drainage

What is a realistic expectation for functional mobility in the community and in the household for a patient with myelodysplasia affecting the midlumbar levels (L3)?

Wheelchair for community mobility Orthoses and walker/crutches for household

Therapeutic intervention Case: Pt is a 35 year old male referred to PT with a dx of R biceps brachii grade II muscle strain. He sustained an injury paying softball 3 weeks ago and was initially placed in a sling for suspected rotator cuff tear. He now has full ROM of RUE but weakness of biceps brachii d/t inactivity (MMT 3+/5). No current pain with ADLs. No PMHx involving RUE. Denies neck pain or any neurologic symptoms. Which set of exercise parameters should be used to initiate a resistance training program for this patient? a. 3 days/wk, 12-15 reps of 3 sets at 50% 1RM b. 7 days/wk, 2-3 reps of 3 sets at 3RM c. 5 days/wk, 10 reps of 3 sets at 10RM d. 3 days/wk, 6-8 reps of 5 sets at 10RM

a. 3 days/wk, 12-15 reps of 3 sets at 50% 1RM

Neuro Case 2: 55 year old female, referred to OP PT secondary to ataxic gait and 3 falls in past 6 months. Reports consistent burning pain, constant numbness/tingling, and weakness in B fingers, ankles, feet, and toes for 1 year. Denies aggravating factors, falls typically happen when in the dark or walking on unlevel surface. PMHx HTN, DM, hypothyroidism, BMI>36, lumbar and B knee OA, B carpal tunnel release 8 years ago. Job as an information technology manager and lives alone. Which of the following exam findings would reinforce both small and large myelinated neural fiber impairments and increased risk for foot ulceration? a. Impaired monofilament and pinprick sensory testing at the foot and toes b. Impaired monofilament and vibration sensory testing at the foot and toes c. Hyporeflexia of the Achilles reflexes and impaired vibration sensory testing of the foot/toes d. Hyperreflexia of the Achilles reflexes and impaired pinprick sensory testing of the foot/toes

a. Impaired monofilament and pinprick sensory testing at the foot and toes

Neuro Case 1: 63 year old male. Pt referred to IP PT secondary to R ischemic stroke with L hemiparesis, admitted 3 days ago with intense HA and LUE>LE weakness exhibits L neglect. PMHx HTN, B knee OA, R knee arthroscopy. He is a college basketball coach, married and lives in a 2-story home. When completing your IE on this pt, you determine they have significant neglect of the L side with complete paresis/sensory loss of the LUE and moderate hypotonia/weakness of the LLE. What initial intervention is the BEST choice for improving this patient's function? a. Midline orientation and safe weight-bearing (approximation) of the involved UE in assisted static sitting b. Constraint induced movement therapy with immobilization of the RUE c. Bilateral scapular stabilization and overhead pulley exercises for LUE ROM d. Dynamic sitting with rapid weight shifts to the L to facilitate a quick stretch and increased tone

a. Midline orientation and safe weight-bearing (approximation) of the involved UE in assisted static sitting

MSK case study 2: pt is 68 year old female, referred to PT with cc of R posterior thigh/knee pain for past 6 weeks, onset after long period of sitting, knee stiffness on waking in morning that lasts <30 min, pain worsens with inactivity and improves with activity, Hx of L knee replacement 2 years ago, DVT ruled out, denies any neuro symptoms or L What are the most common findings in the clinical exam and dx imaging for pts with knee OA a. Hypomobile, painful joint; increased joint space and osteophytes on radiographs b. Hypomobile, painful joint; decreased joint space and osteophytes on radiographs c. Hypermobile, painful joint; increased joint space and osteophytes on radiographs d. Hypermobile, painful joint; decreased joint space and osteophytes on radiographs

b. Hypomobile, painful joint; decreased joint space and osteophytes on radiographs

CV Case: Patient is an 84 year old male, seen in ED for primary complaint of chest tightness, cardiac enzymes were positive for MI and he had surgery for 3-vessel CABG. PT consulted to initiate phase 1 cardiac rehab. PMHx: HTN, HLD, smokes 1 pack/day for 60 yrs. Pt is POD 3. Pt lives alone and uses cane to ambulate, has fallen 3x in last month What would the therapist expect on the initial physical therapy examination? a. Asymmetric breathing pattern b. Increased peripheral edema c. Effective cough d. Normal breath sounds

b. Increased peripheral edema

Geriatric case: pt is a 76 year old male referred to OP PT d/t right knee pain for 3 months after falling in the grocery store, he has fallen 2x in the past year. Knee pain 2/10 on average, increased with stairs or squatting. Xrays show mild OA. Denies numbness, tingling, radiating pain, or weakness in extremities. Patient is having difficulty with short term memory. Suspected AD for 1 year, depression for 6 mos, elevated cholesterol. He is a retired professor and lives with his spouse in 1 story home. What information from the patient or his spouse is most consistent with the diagnosis of mild Alzheimer's dx? a. Decreased attention span and problems recognizing friends and family members b. Increased time required to accomplish daily tasks accompanied with anxiety c. Newly occurring sleep disorders and intermittent hallucinations d. Difficulty with complex planning but daily life not generally affected

b. Increased time required to accomplish daily tasks accompanied with anxiety

Integumentary Case: Patient is a 75 year old male referred for management of woujnd at amputation site. Digits 1-3 on R foot were amputated 2 months ago d/t gangrene and the site has not healed. The pt has been using a silver-based wound gel with a gauze dressing cover changed 2x/day. The wound bed is pale and dry, the skin below the knee is hairless, dry, atrophied, and shiny. Patient reports numbness in legs, feet, and toes. PMHx: 15 year Hx of DM, 20 yr Hx of HTN and PVD, 30 pack years smoker, Raynaud's dx. He is a retired respiratory therapist and BMI=20. Which of the following interventions are important to implement in a pt with a gangrenous toe due to ischemia? a. Hydrogel dressing changed twice a day b. Instruct the pt to keep the limb in a dependent position c. Normothermic hydrotherapy d. Sharp debridement of the gangrenous toe

b. Instruct the pt to keep the limb in a dependent position

MSK case study 1: pt is 49 year old female, referred to OP PT with cc of R sided neck pain, insidious onset 3 days ago, pain and numbness extending to R posterior forearm and dorsum of hand and occasionally to 3rd digit, arm weakness, hx of HTN and occasional HAs with neck pain, school teacher, BMI 34, occasional alcohol consumption During the pt interview, the PT learns that the pt has RA. What special test is most appropriate to determine if patient has upper C spine instability? a. Spurling's b. Sharp Purser c. Quadrant d. Neck flexor muscle endurance

b. Sharp Purser

CV Case: Patient is an 84 year old male, seen in ED for primary complaint of chest tightness, cardiac enzymes were positive for MI and he had surgery for 3-vessel CABG. PT consulted to initiate phase 1 cardiac rehab. PMHx: HTN, HLD, smokes 1 pack/day for 60 yrs. Pt is POD 3. Pt lives alone and uses cane to ambulate, has fallen 3x in last month The first time the pt mobilizes from supine to standing, he demonstrates a drop in BP (supine 128/72, standing 106/68). What is the MOST likely explanation for this response? a. This drop in BP response with position changes is normal and should not be concerning to the PT b. The pt has decreased intravascular volume post-op and is demonstrating orthostatic hypotension c. The workload of the activity was too high, and the pt is unable to maintain their cardiac output d. The lack of standing is causing the BP to drop

b. The pt has decreased intravascular volume post-op and is demonstrating orthostatic hypotension

Neuro Case 2: 55 year old female, referred to OP PT secondary to ataxic gait and 3 falls in past 6 months. Reports consistent burning pain, constant numbness/tingling, and weakness in B fingers, ankles, feet, and toes for 1 year. Denies aggravating factors, falls typically happen when in the dark or walking on unlevel surface. PMHx HTN, DM, hypothyroidism, BMI>36, lumbar and B knee OA, B carpal tunnel release 8 years ago. Job as an information technology manager and lives alone. Which of the following compensatory strategies is BEST to enhance this pt's safety when ambulating in the community? a. Ankle foot orthosis b. Use of cane or walking stick c. Use of a rolling walker d. Use of supportive footwear

b. Use of cane or walking stick

MSK case study 1: pt is 49 year old female, referred to OP PT with cc of R sided neck pain, insidious onset 3 days ago, pain and numbness extending to R posterior forearm and dorsum of hand and occasionally to 3rd digit, arm weakness, hx of HTN and occasional HAs with neck pain, school teacher, BMI 34, occasional alcohol consumption What are the most common findings in clinical exam for pts with C radic? a. (+) ULTT, (-) Spurling's, (-) cervical distraction test, painful ROM b. (+) ULTT, (+) Spurling's, (-) cranial cervical flexion test, painful ROM c. (+) ULTT, (+) Spurling's, (+) cervical distraction test, painful ROM d. (+) ULTT, (+) Spurling's, (+) cranial cervical flexion test, painful ROM

c. (+) ULTT, (+) Spurling's, (+) cervical distraction test, painful ROM

Pulmonary case study: Pt is a 72 year old female, recent 3 day admission with increased SOB and productive cough with yellow-green sputum. Treated with high dose steroids and antibiotics, along with brochodilator for COPD exacerbation. Referred to OP pulmonary rehab 2 weeks after discharge. PMHx: HTN, HLD, DM2, obstructive sleep apnea, obesity, smoking 1-2 packs/day for 45 years. Pt uses CPAP at night, receives assistance from local agency for house cleaning and shopping. Pt lives alone and uses rollator for community ambulation. Which intensity of walking program would be best option for the patient to start in rehab? a. 20% of maximum, 2 days a week, 5 minutes per day b. 30% of maximum, 3 days a week, 10 minutes per day c. 50% of maximum, 5 days a week, 20 minutes per day d. 85% of maximum, 6 days a week, 40 minutes per day

c. 50% of maximum, 5 days a week, 20 minutes per day

Neuro Case 2: 55 year old female, referred to OP PT secondary to ataxic gait and 3 falls in past 6 months. Reports consistent burning pain, constant numbness/tingling, and weakness in B fingers, ankles, feet, and toes for 1 year. Denies aggravating factors, falls typically happen when in the dark or walking on unlevel surface. PMHx HTN, DM, hypothyroidism, BMI>36, lumbar and B knee OA, B carpal tunnel release 8 years ago. Job as an information technology manager and lives alone. When completing your IE on this pt you determine they have significant sensory impairments at the ankles, feet, and toes. They have atrophy of the intrinsic muscles of the feet/toes, but normal strength at/above the ankles. Which of the following impairments in balance/mobility are most consistent with polyneuropathy and this pt's hx and sensory/strength findings? a. Impaired balance with feet apart eyes open resulting in a hip strategy b. Impaired four square step test c. Impaired tandem stance eyes closed requiring a steppage strategy d. Impaired 10 m gait speed that does not improve when the pt looks down

c. Impaired tandem stance eyes closed requiring a steppage strategy

Pulmonary case study: Pt is a 72 year old female, recent 3 day admission with increased SOB and productive cough with yellow-green sputum. Treated with high dose steroids and antibiotics, along with brochodilator for COPD exacerbation. Referred to OP pulmonary rehab 2 weeks after discharge. PMHx: HTN, HLD, DM2, obstructive sleep apnea, obesity, smoking 1-2 packs/day for 45 years. Pt uses CPAP at night, receives assistance from local agency for house cleaning and shopping. Pt lives alone and uses rollator for community ambulation. What is one of the primary benefits of pulmonary rehab for pts with COPD after an exacerbation? a. Decreased dyspnea on exertion b. Decreased hospital admissions c. Improvement in health-related QOL d. Improved mortality rates

c. Improvement in health-related QOL

Therapeutic intervention Case: Pt is a 35 year old male referred to PT with a dx of R biceps brachii grade II muscle strain. He sustained an injury paying softball 3 weeks ago and was initially placed in a sling for suspected rotator cuff tear. He now has full ROM of RUE but weakness of biceps brachii d/t inactivity (MMT 3+/5). No current pain with ADLs. No PMHx involving RUE. Denies neck pain or any neurologic symptoms. Which adjustment should be made to progress the rehab program and increase strength of the affected muscle? a. Increase the number of reps to 10-15 b. Increase the days per week to 5 c. Increase the intensity to 60-70% 1RM d. Increase the number of sets to 5

c. Increase the intensity to 60-70% 1RM

CV Case: Patient is an 84 year old male, seen in ED for primary complaint of chest tightness, cardiac enzymes were positive for MI and he had surgery for 3-vessel CABG. PT consulted to initiate phase 1 cardiac rehab. PMHx: HTN, HLD, smokes 1 pack/day for 60 yrs. Pt is POD 3. Pt lives alone and uses cane to ambulate, has fallen 3x in last month The PT designs a phase 1 walking program for the patient. What is the MOST appropriate program? a. Continuous walking for 20 minutes at 80% of HRmax b. Continuous walking for 30 minutes at 70% of HRm c. Interval walking for a total of 20 mins, 1 minute walk at 60% HRmax, rest 1 minute d. Interval walking for a total of 30 mins, 30 sec walk at 75% HRmax, rest 1 minute

c. Interval walking for a total of 20 mins, 1 minute walk at 60% HRmax, rest 1 minute

Peds Case: Pt is a 12 month old male referred to PT for EI services and dx of down syndrome. He was born at full term via C-section, has a ventricular septal defect MD hopes will close, receiving speech therapy. Child is in foster care, caregiver has many questions, fine motor skills are age appropriate. He can belly crawl, sit independently, get into quadruped, and is starting to pull to stand. Prefers to sit in W and sleep prone with hips abd, ER, and flex. Transitions into sitting from prone by pushing up on hands and max abd hips. Which of the following would be the greatest priority for the PT when planning education with the foster care provider? a. Low muscle tone and potential joint laxity b. Prognosis of gross motor skills, especially ambulation c. Precautions and contraindications associated with AA instability d. Potential causes and interventions for his VSD

c. Precautions and contraindications associated with AA instability

Geriatric case: pt is a 76 year old male referred to OP PT d/t right knee pain for 3 months after falling in the grocery store, he has fallen 2x in the past year. Knee pain 2/10 on average, increased with stairs or squatting. Xrays show mild OA. Denies numbness, tingling, radiating pain, pr weakness in extremities. Patient is having difficulty with short term memory. Suspected AD for 1 year, depression for 6 mos, elevated cholesterol. He is. retired professor and lives with his spouse in 1 story home. The spouse reports his past falls have occurred while he is multitasking or distracted. Which of the following outcome measures would be most helpful to assess the pt's fall risk and mobility? a. Comfortable gait speed b. 5 times sit to stand c. Timed up and go with cognitive task d. Single leg stance eyes open and eyes closed

c. Timed up and go with cognitive task

Other Systems Case: Pt is a 64 year old male referred to OP PT with LBP x 5 years with increased sxs over the past year, no known MOI. Rates pain as a constant 4/10 with no consistent aggravating or easing factors, denies numbness, tingling, or weakness in LEs, denies BB incontinence but reports frequent urination, nocturia, and difficulty starting urination. PMHx: well controll HTN and high cholesterol. He is a married mechanic that does not smoke and rarely drinks. The pt shares that he has noticed periodic blood in his urine, increased pain at night, and ED over the past 6 mos. These symptoms are most consistent with which medical diagnosis? a. prostatitis b. benign prostatic hyperplasia c. prostate cancer d. bladder cancer

c. prostate cancer

Geriatric case: pt is a 76 year old male referred to OP PT d/t right knee pain for 3 months after falling in the grocery store, he has fallen 2x in the past year. Knee pain 2/10 on average, increased with stairs or squatting. Xrays show mild OA. Denies numbness, tingling, radiating pain, pr weakness in extremities. Patient is having difficulty with short term memory. Suspected AD for 1 year, depression for 6 mos, elevated cholesterol. He is. retired professor and lives with his spouse in 1 story home. On your initial eval the pt's gait speed and single leg balance are normal. He is able to complete the TUG with cognitive task in 20 sec with 2 errors. Which of the following evidence based strategies are BEST for safely improving the pt's community mobility and balance? a. Instruct the pt on use of a cane for community ambulation and conduct in clinic gait training with head turns b. Instruct the pt on the use of a RW for community ambulation and conduct reactive postural control training in clinic c. No device or pt education is needed for community ambulation, and the pt should practice the TUG with cognitive task at home with his wife d. Conduct a fall risk assessment and provide pt education and interventions to address identified fall risk factors

d. Conduct a fall risk assessment and provide pt education and interventions to address identified fall risk factors

Peds Case: Pt is a 12 month old male referred to PT for EI services and dx of down syndrome. He was born at full term via C-section, has a ventricular septal defect MD hopes will close, receiving speech therapy. Child is in foster care, caregiver has many questions, fine motor skills are age appropriate. He can belly crawl, sit independently, get into quadruped, and is starting to pull to stand. Prefers to sit in W and sleep prone with hips abd, ER, and flex. Transitions into sitting from prone by pushing up on hands and max abd hips. Which of the following is potentially most problematic with the preferred W sitting? a. Development of kyphosis of the spine b. Further laxity of lateral collateral ligaments in the knees c. Development of bilateral femoral retroversion d. Developing a toeing-in posture when standing

d. Developing a toeing-in posture when standing

Other Systems Case: Pt is a 64 year old male referred to OP PT with LBP x 5 years with increased sxs over the past year, no known MOI. Rates pain as a constant 4/10 with no consistent aggravating or easing factors, denies numbness, tingling, or weakness in LEs, denies BB incontinence but reports frequent urination, nocturia, and difficulty starting urination. PMHx: well controll HTN and high cholesterol. He is a married mechanic that does not smoke and rarely drinks. Which of the following questions is BEST for determination if prostate dysfunction is contributing to the pt's LBP? a. Does the patient have pain in suprapubic region? b. Does the patient have foul smelling urine? c. Does the patient have blood in stool? d. Does the patient have genital and sacral pain?

d. Does the patient have genital and sacral pain?

Neuro Case 1: 63 year old male. Pt referred to IP PT secondary to R ischemic stroke with L hemiparesis, admitted 3 days ago with intense HA and LUE>LE weakness exhibits L neglect. PMHx HTN, B knee OA, R knee arthroscopy. He is a college basketball coach, married and lives in a 2-story home. What impairments and/or activity limitations are most consistent with the pt's presenting symptoms and an acute R (non-dominant hemisphere) MCA stroke? a. LUE flexor synergy b. Non fluent or motor aphasia c. L hemiplegic gait with standby assistance d. Inability to safely roll to the L side

d. Inability to safely roll to the L side

Neuro Case 2: 55 year old female, referred to OP PT secondary to ataxic gait and 3 falls in past 6 months. Reports consistent burning pain, constant numbness/tingling, and weakness in B fingers, ankles, feet, and toes for 1 year. Denies aggravating factors, falls typically happen when in the dark or walking on unlevel surface. PMHx HTN, DM, hypothyroidism, BMI>36, lumbar and B knee OA, B carpal tunnel release 8 years ago. Job as an information technology manager and lives alone. Which of the following health conditions is the most likely cause of the pt's presenting symptoms? a. Lumbar spinal stenosis b. Cervical myelopathy c. Cerebellar disease d. Polyneuropathy

d. Polyneuropathy

Pulmonary case study: Pt is a 72 year old female, recent 3 day admission with increased SOB and productive cough with yellow-green sputum. Treated with high dose steroids and antibiotics, along with brochodilator for COPD exacerbation. Referred to OP pulmonary rehab 2 weeks after discharge. PMHx: HTN, HLD, DM2, obstructive sleep apnea, obesity, smoking 1-2 packs/day for 45 years. Pt uses CPAP at night, receives assistance from local agency for house cleaning and shopping. Pt lives alone and uses rollator for community ambulation. What is the most likely sequelae of pt's medical treatment in the hospital that the PT should consider during their exam? a. Cognitive screening due to ongoing delirium b. Incontinence due to antibiotic use c. Integumentary issue on sacrum due to prolonged immobility d. Proximal weakness due to steroid myopathy

d. Proximal weakness due to steroid myopathy

Integumentary Case: Patient is a 75 year old male referred for management of wound at amputation site. Digits 1-3 on R foot were amputated 2 months ago d/t gangrene and the site has not healed. The pt has been using a silver-based wound gel with a gauze dressing cover changed 2x/day. The wound bed is pale and dry, the skin below the knee is hairless, dry, atrophied, and shiny. Patient reports numbness in legs, feet, and toes. PMHx: 15 year Hx of DM, 20 yr Hx of HTN and PVD, 30 pack years smoker, Raynaud's dx. He is a retired respiratory therapist and BMI=20. What is the most likely reason the wound is not healing? a. Nutritional intake is inadequate b. The pt is not managing their DM properly c. The wound is infected d. There is inadequate blood flow to allow for healing

d. There is inadequate blood flow to allow for healing

Integumentary Case: Patient is a 75 year old male referred for management of woujnd at amputation site. Digits 1-3 on R foot were amputated 2 months ago d/t gangrene and the site has not healed. The pt has been using a silver-based wound gel with a gauze dressing cover changed 2x/day. The wound bed is pale and dry, the skin below the knee is hairless, dry, atrophied, and shiny. Patient reports numbness in legs, feet, and toes. PMHx: 15 year Hx of DM, 20 yr Hx of HTN and PVD, 30 pack years smoker, Raynaud's dx. He is a retired respiratory therapist and BMI=20. Which of the following diagnostic tests is BEST to establish the ideal level for future amputation of this limb? a. ABI b. Capillary refill times c. Semmes-Weinstein monofilament testing d. Transcutaneous oximetry

d. Transcutaneous oximetry

MSK case study 1: pt is 49 year old female, referred to OP PT with cc of R sided neck pain, insidious onset 3 days ago, pain and numbness extending to R posterior forearm and dorsum of hand and occasionally to 3rd digit, arm weakness, hx of HTN and occasional HAs with neck pain, school teacher, BMI 34, occasional alcohol consumption What intervention is BEST choice for reducing pain and improving function in acute C radic? a. transcutaneous electrical stimulation (TENS) b. cryotherapy c. thoracic manipulation d. cervical mobilizing and stabilizing techniques

d. cervical mobilizing and stabilizing techniques

MSK case study 2: pt is 68 year old female, referred to PT with cc of R posterior thigh/knee pain for past 6 weeks, onset after long period of sitting, knee stiffness on waking in morning that lasts <30 min, pain worsens with inactivity and improves with activity, Hx of L knee replacement 2 years ago, DVT ruled out, denies any neuro symptoms or L What interventions offer strong evidence for reducing pain and improving function in OA? a. TENS b. creyotherapy c. therapeutic ultrasound d. therapeutic exercise (NM and functional)

d. therapeutic exercise (NM and functional)

Compare and contrast the requirements for supervision of the PTA and PT aide in the provision of care

-PTA operates under the direction of the PT who must be available at least by telecommunications -PT aide operates under the direct, on site supervision of the PT or PTA

What are the documentation requirements for time-based billing for Medicare beneficiaries?

-Time in and time out or total time -Services described by timed codes, untimed codes, and unattended activities -Total number of units billed is limited by the total minutes of the 1 on 1 encounter

A patient has been referred for PT post spinal cord injury due to trauma to the thoracic spine. On assessment, the patient is found to have loss of bilateral motor function as well as pain and temperature sensation. Which of the following is the MOST appropriate diagnosis in this case? A. Anterior cord syndrome B. Posterior cord syndrome C. Central cord syndrome D. Hemi cord syndrome

A. Anterior cord syndrome

A 67-year-old patient had total hip replacement 1 week ago reports a new onset of calf pain. Which of the following findings shouldbe reported to the physician immediately in suspicion of DVT? A. Bed ridden for more than 3 days, history of DVT, calf swelling more than 3mm on asymptomatic side and active cancer B. History of hip arthritis, swelling around the hip joint C. B/L pitting edema and painful straight leg raises D. Trochanteric bursitis, painful hip internal rotation and swelling on lateral side of hip

A. Bed ridden for more than 3 days, history of DVT, calf swelling more than 3mm on asymptomatic side and active cancer

A patient was admitted to a hospital after a skiing accident which resulted in a spinal cord injury. While doing a review of the patient's medical charts, the PT reads that the patient had an episode of autonomic dysreflexia a few days ago. Which of the following sign/symptom is MOSTLY associated with this presentation? A. Bradycardia and hypertension B. Hypotension and bradycardia C. Tachycardia and hypertension D. Hypotension and tachycardia

A. Bradycardia and hypertension

A patient has a grade III pressure ulcer on the hip. The wound has excessive amounts of exudate present. Which of the following is the MOST appropriate dressing to use? A. Calcium alginate B. Hydrocolloid dressing C. Hydrogel dressing D. Transparent film

A. Calcium alginate

A 56-year-old patients reports having difficulty in cervical extension due to a closing restriction. The manual therapy technique appropriate to correct a bilateral closing restriction of C5 on C6 is: A. Central PA glide on the spinous process of C6 while stabilizing C5. B. Central PA glide on the spinous process of C5 while stabilizing C6. C. Unilateral PA glide on the right transverse process of C5 while stabilizing C6. D. Unilateral AP glide on the left transverse process of C6 while stabilizing C5.

A. Central PA glide on the spinous process of C6 while stabilizing C5.

The manual therapy technique appropriate to correct a bilateral closing restriction of C5 on C6 is: A. Central PA glide on the spinous process of C6 while stabilizing C5. B. Central PA glide on the spinous process of C5 while stabilizing C6. C. Unilateral PA glide on the right transverse process of C6 while stabilizing C5. D. Unilateral AP glide on the left transverse process of C6 while stabilizing C5.

A. Central PA glide on the spinous process of C6 while stabilizing C5.

A PT instructed a patient in home postural drainage positioning. The PT finds the patient siting in a recliner and leaning forward trying to perform drainage for the superior segments of the lower lobes. What is the best way for the PT to correct the position? A. Change the position to prone lying on a bed with two pillows under the pelvis B. Keep sitting in the recliner, leaning forward over a folded pillow C. Change the position to supine on a bed with pillows under the knees D. Keep sitting in a recliner, leaning slightly backward

A. Change the position to prone lying on a bed with two pillows under the pelvis

Which of the following is MOST likely to cause a dislocation after a total hip arthroplasty with an anterior approach? A. Combined motion of hip flexion, abduction, external rotation B. Sitting only on high chairheight C. Performing a step-to ambulation during the opposite side swing phase D. Leading with the affected leg while going downstairs

A. Combined motion of hip flexion, abduction, external rotation

A PT is examining a 55-year-old patient who has been referred for rehabilitation post traumatic brain injury. During assessment which of the following signs/symptoms are LEAST likely to be present in this case? A. Hypotonia and decreased reflexes B. Hypertonicity in the affected muscles C. Positive Babinski sign D. Presence of clonus with rapid passive foot dorsiflexion

A. Hypotonia and decreased reflexes

A physical therapist is working with a 53-year-old male retired physician. The patient had a stroke that affected his anterior cerebral artery. Which of the following presentations is the patient MOST likely going to display? A. Contralateral hemiparesis of LE, urinary incontinence, apraxia, and contralateral hemisensory loss of LE B. Contralateral hemiparesis of UE and face, contralateral hemiparesis of UE, global aphasia, and ataxia C. Contralateral homonymous hemianopia, prosopagnosia, visual agnosia, and memory defect D. Ipsilateral paralysis of conjugate gaze, nystagmus, diplopia on lateral gaze, and paresis of face, UE, and LE.

A. Contralateral hemiparesis of LE, urinary incontinence, apraxia, and contralateral hemisensory loss of LE

A home health PT goes to a patient's house for initial evaluation. The patient has been diagnosed with C7 level ASIA B injury. During inspection, the PT notes that the patient has partial thickness skin loss on the sacral region. The wound barely has an exudate and there are no signs of infection. There is no slough present. Which of the following is the MOST appropriate set of interventions in this case? A. Cover wound with transparent films and teach patient to independent pressure relief every 2 hour B. Cover wound with hydrogel and tell the patient to remind the family to do skin inspection and assist with pressure relief every 2 hours C. Cover the wound with alginate dressing and inform the physician about the finding and document it D. Cover the wound with Iodine-based dressing and ask the patient go to urgent care for further inspection.

A. Cover wound with transparent films and teach patient to independent pressure relief every 2 hour

A patient is suspected to have insufficiency of adrenal glands causing reduced secretion of cortisol. Which of the following signs are MOST likely expected to be present in this case? A. Dark pigmentation of the skin and weight loss B. Dark pigmentation of the skin and weight gain C. Osteoporosis and weight gain D. Osteoporosis and weight loss

A. Dark pigmentation of the skin and weight loss

A PT is performing phonophoresis at 0.8 W/cm2, 1.0 MHz and 20% duty cycle on the distal hamstrings. During treatment, the patient reports feeling pain over distal hamstrings. The PT's FIRST appropriate course of action is to: A. Decrease the intensity to 0.5 W/cm. and continue moving the head at same speed B. Keep the same intensity at 0.8 W/cm.and change the setting to 50% duty cycle C. Increase the intensity to 2.0 W/cm. and add more coupling medium D. Turn off the ultrasound and immediately call the physician

A. Decrease the intensity to 0.5 W/cm. and continue moving the head at same speed

A patient presents with chief concerns of knee pain. On assessing the patient, the PT notes that the patient has some discomfort while flexing the right knee. Which of the following is the MOST appropriate direction to give a glide to increase knee flexion? A. Distal glide to patella in supine B. Proximal glide to patella in supine C. Compression of the patella in supine D. Anterior glide to tibia on a stable femur in prone.

A. Distal glide to patella in supine

After evaluating a baseball player, the PT concludes that there is insufficient deceleration of the shoulder during throwing. Which of the following strengthening techniques will beMOST effective in improving control of deceleration of the shoulder? A. Eccentric exercises of the teres minor and infraspinatus B. Eccentric exercises of the teres major and pectoralis major C. Concentric exercises of the teres minor and infraspinatus D. Concentric exercises of the teres major and pectoralis major

A. Eccentric exercises of the teres minor and infraspinatus

Which of the following activities should be the PRIMARY emphasis of a PT treatment program for a child who has athetoid cerebral palsy? A. Facilitating co-contraction patterns and encouraging control in voluntary movement B. Preservation of strength and muscle tone C. Facilitating use of primitive reflexes to perform fine motor skills D. Prevention of contractures and determine the best method of mobility

A. Facilitating co-contraction patterns and encouraging control in voluntary movement

A 82-year-old patient suffered a right sided CVA about a week ago. The therapist is educating him on various positioning strategies. Which of the following is the MOST appropriate while lying on the left side? A. Head/neck: neutral, left Scapular protracted; L arm in slight abduction and external rotation; elbow extended, forearm supinated, wrist neutral, fingers extended, and thumb abducted. B. Head/neck: neutral, left Scapular retracted; L arm in slight abduction and internal rotation; elbow extended, forearm pronated, wrist neutral, fingers extended, and thumb adducted C. Head/neck: neutral, right Scapular retracted; L arm in slight abduction and internal rotation; elbow extended, forearm pronated, wrist neutral, fingers extended, and thumb adducted D. Head/neck: neutral, left Scapular protracted; L arm in slight adduction and external rotation; elbow flexed, forearm supinated, wrist extended, fingers flexed, and thumb abducted.

A. Head/neck: neutral, left Scapular protracted; L arm in slight abduction and external rotation; elbow extended, forearm supinated, wrist neutral, fingers extended, and thumb abducted.

A PT is considering implementing electrical stimulation in the patient's plan of care to promote healing during proliferative phase of a non infected wound. Which of the following will be MOST effective for this patient? A. High voltage pulsed waveform on positive electrode in wound, 110 PPS. B. High voltage pulsed waveform on negative electrode in wound, 10 PPS. C. High voltage pulsed waveform on positive electrode in wound, 10 PPS. D. High voltage pulsed waveform on negative electrode in wound, 110 PPS.

A. High voltage pulsed waveform on positive electrode in wound, 110 PPS.

A therapist is training a 65-year-old to walk with a quadripod. He had severe arthritis of the R hip and was operated 2 months ago. What is the MOST appropriate way the PT should instruct the patient to use the assistive device? A. Hold the cane in your L hand and move L hand and R leg together B. Hold cane in your R hand and move R hand and L leg together C. Hold cane in your L hand and move L hand and L leg together D. Hold cane in your R hand and move R hand and R leg together

A. Hold the cane in your L hand and move L hand and R leg together

A patient suffered from a left hemisphere CVA few weeks ago. The PT instructed patient to pick up an object and patient did not pick it. The PT stepped away within view from the patient and noticed him picking up the object. Which of the following MOST LIKELY describes this scenario? A. Ideomotor apraxia B. Ideational apraxia C. Aphasia D. Ataxia

A. Ideomotor apraxia

A 30 years old male client visits a town which is 9000 feet above sea level. What are the INITIAL cardiovascular responses during the first week in town? A. Increased BP, increased cardiac output with tachycardia and no significant changes in SV. B. Decreased BP, decreased cardiac output with bradycardia and increased SV. C. Increased BP, decreased cardiac output with bradycardia and increased SV. D. Decreased BP, increased cardiac output with tachycardia and increased SV.

A. Increased BP, increased cardiac output with tachycardia and no significant changes in SV.

A patient is referred with a diagnosis of chronic periarthritis of the shoulder. The patient feels tired all the time and has noticed recent weight loss in spite of an increased appetite. Which of the following would be seen along with these symptoms if his thyroid levels are elevated? A. Increased DTR, increased HR, low BP, heat intolerance B. Decreases DTR, increased HR, high BP, cold intolerance C. Increased DTR, increased HR, low BP, cold intolerance D. Decreases DTR, increased HR, high BP, heat intolerance

A. Increased DTR, increased HR, low BP, heat intolerance

A 69-year-old patient has symptoms of lightheadedness, along with pain and cramping of arms and legs. The patient is hyperventilating, and the blood gas report reads: pH-7.48, PaCO2-32 mm Hg and HCO3-25 mEq/L Which of the following is the LEAST likely treatment? A. Intravenous injection of sodium bicarbonate B. CO2 administration C. Pain control D. Using a Rebreathing mask

A. Intravenous injection of sodium bicarbonate

Which of the following are the two hallmark signs associated with right-sided heart failure? A. Jugular venous distension and peripheral edema B. Dyspnea and cough C. Dyspnea and pulmonary edema D. Dyspnea and orthopnea

A. Jugular venous distension and peripheral edema

A 20-year-old reports of pain, stiffness and feeling of weakness in his left knee which aggravates after playing. There is no crepitus, no popping or locking of the joint. Palpating the lower pole of patella and quadriceps tendon reproduces pain. What is MOST likely diagnosis? A. Jumpers knee B. Meniscal tear C. Osteochondritis dissecans D. Osgood Schlatter syndrome

A. Jumpers knee

A PT is assessing a patient with scoliosis. Which of the following deformities is MOST likely to be associated with right-sided thoracic scoliosis? A. Left anterior rib hump and right posterior rib hump B. Right anterior rib hump and left posterior rib hump C. Right anterior and posterior rib hump D. Left anterior and posterior rib hump

A. Left anterior rib hump and right posterior rib hump

A patient with Angina Pectoris has been advised to use Sublingual NTG for stable angina. This drug will MOST LIKELY cause which of the following? A. Lower preload and afterload B. Increase myocardial oxygen demand C. Increase preload and afterload D. Hypertension and bradycardia

A. Lower preload and afterload

A patient with T-score of -2.6 and no fractures complains of increased middle and lower back pain during breathing and other functional activities. Which of the following is MOST beneficial intervention? A. Lumbar extension and core stabilization exercises. B. Lumbar rotation and core stabilization exercises. C. Lumbar rotation and extension exercises. D. Lumbar flexion and rotation exercises.

A. Lumbar extension and core stabilization exercises.

A PT is treating a 22-year-old male patient who has been classified as ASIA C. Which of the following is TRUE about this patient? A. Motor function is preserved with only 1/3 of the key muscles below the neurological level of injury have a muscle grade of less than 3. B. Motor function is preserved with at least 1/2 of the key muscles below the neurological level of injury have a muscle grade of more than 3. C. Motor function is preserved with all of the key muscles below the neurological level of injury have a muscle grade of more than 3. D. Motor function is preserved with more 1/2 of the key muscles below the neurological level of injury have a muscle grade of less than 3

A. Motor function is preserved with only 1/3 of the key muscles below the neurological level of injury have a muscle grade of less than 3.

A patient comes to PT clinic following a left below knee amputation. Assuming the black dot in the middle was the patient's COM before amputation, which of the following location is MOST likely to be the new COM after amputation. A. Moves higher on the right side (Yellow dot) B. Moves lower on the right side (Red dot) C. Moves higher on the left side (Blue dot) D. Moves lower on the left side (Green dot)

A. Moves higher on the right side (Yellow dot)

A PT is treating a patient who was originally diagnosed with a pressure injury stage 3. The patient presents to clinic with ulcer shown in the picture. Which of the following findings is MOST accurate? A. No change has occurred in the pressure injury B. The pressure injury has improved to Stage 1 C. The pressure injury has improved to Stage 2 D. The pressure injury has become unstageable

A. No change has occurred in the pressure injury

An Intrathecal Baclofen pump has been put in for a child with spastic CP. The therapist observes that the child is very drowsy, tone appears floppy and is not breathing well. What is the MOST likely cause of the above symptoms ? A. Overdose of Baclofen B. Child is experiencing fatigue C. Dosage of Baclofen is less D. Child is not interested in therapy

A. Overdose of Baclofen

A 52-year-old patient presents to the clinic with chronic LBP. After mobility assessment of the lumbar spine, PT concludes that patient as an opening restriction at right L2-L3 level. Which of the following should the PT implement in the treatment ? A. PA glide to right L2 transverse process B. PA glide to left L2 transverse process C. PA glide to L3 spinous process D. PA glide to L2 spinous process

A. PA glide to right L2 transverse process

A patient complains of reduced mobility in the right hip. PT notices that the patient has decreased hip flexion with the knee in extension, but hip flexion increases when the knee is flexed. What is the MOST likely cause of reduced hip flexion with knee extension ? A. Passive insufficiency of hamstrings B. Passive insufficiency of quadriceps C. Active insufficiency of hamstrings D. Passive insufficiency of iliacus

A. Passive insufficiency of hamstrings

A PT Aide was treating a patient who had a recent CVA and asked the patient to complete bridging exercises that were not in the original POC. During the bridging exercises, the patient suffered a back injury. Who will need to take the responsibility for the injury? A. Physical therapist B. Physical therapy assistant C. Physical therapy aide D. Physical therapy student

A. Physical therapist

A 28-year-old female patient with Above Knee Amputation (AKA) presents with a hyper flexed knee joint on the prosthetic limb. The PT can suggest the prosthetist to make the following modifications: A. Plantarflex the foot B. Move the foot laterally C. Move the foot medially D. Dorsiflex the foot

A. Plantarflex the foot

A patient with BMI of 33 kg/m2 presents to a clinic with pain and swelling on the right hip. The therapist is planning to use ultrasound as a therapeutic modality. Which of the following is an ABSOLUTE contraindication for use of ultrasound? A. Polyethylene hip implant B. Chronic right hip fracture C. Metal hip implant D. Acute hip adductor tear

A. Polyethylene hip implant

A PT is working with a patient who has cystic fibrosis. The PT intends on using postural drainage as a treatment. PT positions the patient in the seated position and has the patient leaning over a pillow. Based on the position of the patient which lobe is MOST likely being drained? A. Posterior apical segments of upper lobe B. Anterior apical segments of upper lobes C. Superior segments of the lower lobes D. Anterior basal segments of lower lobes

A. Posterior apical segments of upper lobe

A PT is developing a treatment plan for a 32-year-old male patient 4 weeks post arthroscopic debridement of the L knee joint. The PT decides to use FES to improve the strength of the quadriceps. Which of the following parameters is the MOST APPROPRIATE? A. Pulse duration-250-300 microsec; 10 seconds on, 50 seconds off B. Pulse duration-150-170 microsec; 10 seconds on, 20 seconds off C. Pulse duration-250-300 microsec; 10 seconds on, 20 seconds off D. Pulse duration-150-170 microsec; 10 seconds on, 50 seconds off

A. Pulse duration-250-300 microsec; 10 seconds on, 50 seconds off

A PT is developing a treatment plan for a patient who has a 10 deg of extension lag. The PT decides to use electrical stimulation to improve the strength of the quadriceps. Which of the following parameters is the MOST APPROPRIATE for this patient? A. Pulse duration-270 microsec; 8 seconds on, 40 seconds off; Pulse frequency-60-70 pps B. Pulse duration-150 microsec; 8 seconds on, 16 seconds off; Pulse frequency-60-70 pps C. Pulse duration-250 microsec; 8 seconds on, 16 seconds off; Pulse frequency-60-70 pps D. Pulse duration-150 microsec; 8 seconds on, 40 seconds off; Pulse frequency-60-70 pps

A. Pulse duration-270 microsec; 8 seconds on, 40 seconds off; Pulse frequency-60-70 pps

A two-year-old child with Down's syndrome is being treated by a physical therapist. The child has moderate developmental delay and exhibits hypo tonicity. The MOST appropriate physical therapy treatment should include: A. Pushing a toy cart while standing B. Locomotor training using body weight support C. Rolling and somersault activities D. Rhythmic stabilization of postural extensors in sitting

A. Pushing a toy cart while standing

A 32-year-old female presents to the clinic 5 weeks post Colle's fracture following a fall. She demonstrates full finger flexion and extension but limited 1st CMC extension. Which of the following mobilizations is most appropriate to perform? A. Radial glide of the MC on the trapezium B. Ulnar glide of the MC on the trapezium C. Anterior glide of the MC on the trapezium D. Posterior glide of the MC on the trapezium

A. Radial glide of the MC on the trapezium

A 20-year-old patient in ROTC complains of anterior hip pain. During treatment, the patient starts flirting with the male PT. The PT acts professionally but the patient persists and makes the PT uncomfortable. What would be MOST appropriate response? A. Refuse to treat the patient and refer to another male PT B. Ignore the flirting and continue to treat C. Notify patient's ROTC supervisor D. Joke along to keep open the possibility of dating in the future

A. Refuse to treat the patient and refer to another male PT

A 20-year-old patient in Reserve Officer Training Corps (ROTC) complains of anterior hip pain. During treatment, the patient starts flirting with the male PT. The PT acts professionally but the patient persists and makes the PT uncomfortable. What would be MOST appropriate response? A. Refuse to treat the patient and refer to another male PT B. Ignore the flirting and continue to treat C. Notify patient's ROTC supervisor D. Joke along to keep open the possibility of dating in the future

A. Refuse to treat the patient and refer to another male PT

A patient has sudden onset of symptoms of slurred speech, and unable to pronounce the words properly. The patient has tongue deviation as shown in the picture below. Which of the following is the MOST LIKELY cause for this? A. Right Hypoglossal nerve palsy B. Left Glossopharyngeal nerve palsy C. Left Hypoglossal nerve palsy D. Right Glossopharyngeal nerve palsy

A. Right Hypoglossal nerve palsy

A patient complains of sudden onset of mild hearing loss on the right side. Rinne test was consistent with air conduction greater than bone conduction on both sides. Weber's test findings show sound lateralized to the left ear. Which of the following is the MOST likely? A. Right side sensorineural hearing loss B. Left side conduction hearing loss C. Right side conduction hearing loss D. Left side sensorineural hearing loss

A. Right side sensorineural hearing loss

A patient presents to the clinic with chief complains of difficulty opening jars and turning doorknobs. There is some weakness of the intrinsic muscles along with wasting of the hypothenar eminence. Which of the following test will MOST LIKELY be positive? A. Shoulder abduction and external rotation with elbow flexion and supination, wrist extension and radial deviation, finger extension B. Shoulder abduction and medial rotation with elbow extension and pronation, wrist and finger flexion with contralateral cervical side flexion C. Shoulder abduction with elbow and wrist extension, forearm supination and finger extension. D. Shoulder abduction and lateral rotation with elbow, wrist and finger extension and forearm supination

A. Shoulder abduction and external rotation with elbow flexion and supination, wrist extension and radial deviation, finger extension

A PT is creating an exercise program for a patient who underwent an anterior approach total hip arthroplasty 3 weeks ago. Which of the following exercises would MOST safely promote gluteal strengthening? A. Sit to stand transfer exercises B. Standing hip extension exercises C. Standing external rotator strengthening using a theraband D. Sitting external rotator strengthening using an ankle weight

A. Sit to stand transfer exercises

A 44-year-old female patient arrived at a clinic after sustaining a fall involving a hand/wrist injury. Upon examination, the PT observed the radius dislocated in a volar direction. Which of the following conditions would MOST likely match this description? A. Smith's fracture B. Colle's fracture C. Scaphoid fracture D. Dinner fork deformity

A. Smith's fracture

A PT is treating a patient and is working on weight shifts and reaching activities. The PT asks the patient to place the L hand on his L thigh and use the R hand to reach diagonally across the body and touch the target. The patient has difficulty following the instructions and performs the task incorrectly. Which of the following is the MOST probable diagnosis in this case? A. Somatoagnosia B. Prosopagnosia C. Depth perception disorder D. Tactile Agnosia

A. Somatoagnosia

A patient with an ASIA D T7 vertebral level injury is in inpatient rehabilitation. He asks his physical therapist what to expect with their bladder control. Which of the following statements MOST accurately classifies the bladder dysfunction as well as treatment needed for this patient? A. Spastic urinary dysfunction, Suprapubic tapping B. Flaccid urinary dysfunction, Suprapubic tapping C. Spastic urinary dysfunction, Crede'smaneuver D. Flaccid urinary dysfunction, Valsalva maneuver

A. Spastic urinary dysfunction, Suprapubic tapping

A 39-year-old comes to an outpatient clinic complaining of hip problems. The physical therapist notices a drop of the left hip during right midstance. The MOST appropriate treatment for this impairment would be: A. Stand on right leg and abduct the left leg B. Stand on left leg and extend the right leg C. Stand on right leg and flex the right leg D. Stand on left leg and flex the right leg

A. Stand on right leg and abduct the left leg

A group of PT students are compiling an injury prevention program for adolescent soccer players. The p-value is set at 0.05. The critical value for rejecting the null hypothesis is 1.45, and the statistical test result is 1.85. Which of the following statement is correct? A. Statistically significant, and the probability that the observed sample mean could have occurred by chance is less than 5%. B. Statistically significant, and the probability that the observed sample mean could have occurred by chance is greater than 10%. C. Not statistically significant, and the probability that the observed sample mean could have occurred by chance is 10%. D. Not statistically significant, and the probability that the observed sample mean could have occurred by chance is greater than 50%.

A. Statistically significant, and the probability that the observed sample mean could have occurred by chance is less than 5%.

After normal delivery, the woman comes to a PT clinic with a complaint of leaking of urine involuntarily during coughing and sneezing and when she is climbing stairs. Which of the following diagnosis is MOST likely based on the patient presentation? A. Stress incontinence B. Overflow incontinence C. Functional incontinence D. Urge incontinence

A. Stress incontinence

A patient has just started using Ventolin for management of his condition. What is the patient MOST likely to experience as a side effect of the drug? A. Tachycardia B. Hypotension C. Lightheadedness D. Increased risk of falls

A. Tachycardia

A physical therapist performed a balance test using a new equipment on a healthy individual. The same PT repeated the same test following a week after initial assessment. Which is the most important factor responsible for the success of this assessment? A. Test-retest reliability B. Intra-rater reliability C. Inter-rater reliability D. Content validity

A. Test-retest reliability

A 45-year-old male with a BMI of 38 kg/m2is enrolled in a 6-week fitness training program. Which is the MOST appropriate measure to assess change in fitness from pre and post fitness training? A. The time it takes for the heart rate to return to baseline B. Resting respiration rate at pre training C. Rating on a Wong Baker scale D. Increase in blood pressure during exercise

A. The time it takes for the heart rate to return to baseline

A PT is examining a 46-year-old patient who underwent a radical mastectomy. The patient has developed lymphedema on the right UE. Which of the following statements is the CORRECT intervention for this patient? A. The trunk and axilla should be decongested first followed by arm and hand B. Manual lymphatic drainage should involve proximal to distal stroking C. The hand should be decongested first followed by trunk and axilla D. Manual lymphatic drainage is contraindicated in post mastectomy

A. The trunk and axilla should be decongested first followed by arm and hand

A 50-year-old male patient presents to an outpatient clinic with complains of medial knee pain. The picture represents the position of the foot during normal quiet standing. What is the MOST appropriate diagnosis of this condition? A. Uncompensated forefoot valgus B. Compensated forefoot valgus C. Uncompensated forefoot varus D. Compensated forefoot varus

A. Uncompensated forefoot valgus

The PT is reviewing the lab reports of a patient admitted in an ICU. The BEST diagnosis for this patient would be: A. Uncompensated metabolic acidosis. B. Uncompensated respiratory alkalosis. C. Uncompensated respiratory acidosis. D. Uncompensated metabolic alkalosis.

A. Uncompensated metabolic acidosis.

A physical therapist is examining a 41-year-old male patient in an outpatient clinic. Which of the following sound is PT auscultating? A. Vesicular sounds B. S1 sound C. S2 sound D. Tracheal sounds

A. Vesicular sounds

A PT uses CTSIB to test standing balance of a patient. The patient demonstrates increased sway and experiences a fall during condition 4. Which sensory input system could the PT suspect to be impaired? A. Vision B. Somatosensory C. Vestibular D. Somatosensory and vision

A. Vision

A 45-year-old patient been walking 3 days/week for 20 minutes for the past 3 weeks. When progressing the exercise program, which of the following modifications will MOST likely accomplish the weight-loss goal? A. Walk 4 days/week at current walking speed and increase the duration to 45 minutes B.Increase the walking speed and keep the duration at 20 minutes. C. Walk 6 days/week and decrease the duration to 10 minutes. D. Decrease the walking speed and increase the duration at 25 minutes.

A. Walk 4 days/week at current walking speed and increase the duration to 45 minutes

A 74-year-old retired contractor walks to the treatment room with a prominent backward lean, when he is in stance on the left lower extremity. From this observation, you hypothesize that the MOST LIKELYcause is: A. Weak hip extensors on the left during stance phase B. Weak hip flexors on the left during stance phase C. Weak hip extensors on the right during swing phase D. Weak hip flexors on the right during swing phase

A. Weak hip extensors on the left during stance phase

In a research study on obese THA patients, a correlation coefficient (r) of +0.80 was found for the relationship between weight and BMI. Which of the following interpretations of this finding is MOST appropriate? A. Weight and BMI have a high positive correlation. B. 80 percent of the variability in BMI can be accounted for by weight C. 80 percent of the variability in weight can be accounted for by BMI D. There are no significant differences between the weight and BMI levels

A. Weight and BMI have a high positive correlation.

A PT is assessing a 65-year-old patient and asks the patient how he is doing. The patient's response is, "the sun is yellow, and the water is cold". Which of the following is the MOST appropriate diagnosis of this finding? A. Wernicke's aphasia B. Broca's aphasia C. Non-fluent aphasia D. Lesion to CN VII

A. Wernicke's aphasia

A patient has a superficial partial-thickness wound with 90% necrosis and 10% granulation tissue. Which of the following is MOST appropriate wound care option? A. Wet to dry dressings B. Sharp debridement C. Enzymatic debridement D. Autolytic debridement

A. Wet to dry dressings

A 61-year-old female presents with a wound on her right lower extremity and has a history of painful cramping in the legs especially after walking for a few minutes. Medical history is significant for DM II and hypertension. Which of the following would BEST describe the characteristic of this wound? A. Wound located on the dorsum of toes, base of the wound is pale and necrotic with lack of granulation tissue B. Wound located on the dorsum of the foot, hemosiderin staining present along with fibrosis of dermis C. Wound located on the medial malleolus with swelling of bilateral lower extremities that is relieved with rest D. Pitting edema in the lower extremities, numbness and tingling along with hyperkeratosis of the skin

A. Wound located on the dorsum of toes, base of the wound is pale and necrotic with lack of granulation tissue

A patient presents with an acute sprain of the right ankle. According to the patient, this has occurred frequently over the past 5 years. The clinical test that the PT should use to examine the laxity of the anterior talofibular ligament is A.Anterior drawer test B.Talar tilt C.Thompson's test D.Morton's test

A.Anterior drawer test

A 30-year old male patient presents with right lower abdominal pain. PT performs the "pinch-an-inch" test result is positive. Which condition below is MOSTLY related to the symptoms described? A.Appendicitis B.Diverticulitis C.Crohn's disease D.Irritable bowel syndrome

A.Appendicitis

A patient visits an outpatient clinic with pain in his right scapula, and right shoulder. He states that pain intensifies with ingestion of fatty food. What is the MOST likely diagnosis? A.Cholecystitis B. Irritable bowel syndrome C.HiatalHernia D.GERD

A.Cholecystitis

A 19 year oldpatient is being seen in PT after sustaining an injury doing hurdles a week and a half ago. She reports she felt a pop and instant pain and loss of function in the R hip. She reports pain with walking, tenderness when sitting, and she lacks terminal knee extension during gait. Which of the following treatment plans is most appropriate for this session? A.Ice and estim, education on AC use, RICE, and gentle core activation B.Submaxisometrics, cross friction massage, gentle hip stretching C.Single leg standing and balance activities to regain WBingtolerance, submaxisometrics, cross friction massage D.RICE, pelvic tilts, gentle hip stretching

A.Ice and estim, education on AC use, RICE, and gentle core activation

Which of the following coupling motions will close the right lumbar facet joints from a neutral position? A.Lumbar extension, right side flexion, left rotation B.Lumbar extension, left side flexion, right rotation C.Lumbar flexion, right side flexion, left rotation D.Lumbar flexion, left side flexion, right rotation

A.Lumbar extension, right side flexion, left rotation

A patient presents with foot pain during running. The PT noted the patient has excessive foot pronation and would like to provide an orthotic that could help relieve pain. Which of the following would MOST LIKELY benefit this patient? A.Medial post under first metatarsal head B.Cushion heel C.Lateral post under fifth metatarsal head D.Posterior leaf spring

A.Medial post under first metatarsal head

A patient has been diagnosed with C8 ASIA A injury. Which of the following is MOST appropriate about the clinical presentation of this patient? A.Patient will be able to transfer independently on level surfaces B. All sensation will be present at S4-S5 level C. Patient will not be able to perform elbow flexion and wrist extension D. Some motor function will be present at S4-S5 level

A.Patient will be able to transfer independently on level surfaces

A patient presents to the clinic with chief complaints of left sided neck pain that started about 2 weeks ago. After assessing, the PT concludes that the patient has a closing restriction at C5-C6 level on the left. Which of the following is MOST appropriate about this patient? A.Side bending and rotation to the left is restricted at C5-C6 level B.Side bending and rotation to the right is restricted at C5-C6 level C.Side bending is restricted to right, and rotation is restricted to the left at C5-C6 level D.Side bending is restricted to the left and rotation is restricted to the right at C5-C6 level

A.Side bending and rotation to the left is restricted at C5-C6 level

A PT is evaluating a 26-year-old female basketball player with a vague diagnosis of right knee pain. The patient is performing a jump landing as shown. The PT treatment should MOST likely focus on: A.Strengthening of the right gluteus medius B.Strengthening of the right adductor magnus C.Strengthening of the right vastus medialis D.Strengthening of the right tibialis anterior

A.Strengthening of the right gluteus medius

A PT is treating a patient who has lymphedema in the right LE and BP at 135/85 mm Hg. The PT uses mechanical, intermittent pneumatic compression using a sleeve that covers the LE. At first treatment session, the MOST appropriate inflation pressure is? A. 10 mm Hg B. 60 mmHg C. 100 mmHg D. 140 mmHg

B. 60 mmHg

A 75-year-old individual has limited endurance. There is no history of cardiorespiratory problems. After an exercise tolerance test, which was negative for coronary heart disease, the BEST initial exercise prescription for this individual would be: A. 90-100% HR max. B. 60-70% HR max. C. 40-50% HR max. D. 30-40% of HR max.

B. 60-70% HR max.

A child is being treated for PT in an outpatient clinic. The therapist notices that the child is unable to transfer a toy from one hand to other hand. What age would be considered normal for a child to be able to transfer an object from one hand to another hand? A. 4 months B. 9 months C. 12 months D. 18 months

B. 9 months

Which of the following is NOT an expected sign seen in a patient with facial nerve (CN VII) palsy. A. Hyperacusis B. Absence of sensation in the anterior 2/3 of the tongue C. Absence of corneal reflex D. Decreased lacrimation

B. Absence of sensation in the anterior 2/3 of the tongue: done by Trigeminal Facial is for TASTE (not sensation) on anterior 2/3 tongue

A physical therapist is working with a patient who has tuberculosis, Which of the following options BEST describes the appropriate precautions and type of personal protective equipment that a physical therapist should wear? A. Contact precautions, N-95 respirator B. Airborne precautions, N-95 respirator C.Contact precautions, Gloves and Gown D. Airborne precautions, Gloves and Gown

B. Airborne precautions, N-95 respirator

A patient presented with C6 SCI from a fall causing hyperflexion of the cervical spine. The PT observed patient had weak wrist and finger flexion, decreased sense of pain and temperature. However, patient's light touch and vibratory sense were preserved. Which of the following is MOST likely associated with this scenario? A. Central Cord Syndrome B. Anterior Cord Syndrome C. Brown-Sequard Syndrome D. Posterior Cord Syndrome

B. Anterior Cord Syndrome

A PT is training a 21-year-old male who has suffered TBI. The patient is being trained to perform sit-to-stand transfer with verbal and tactile feedback from the PT. Which of the following order of practice is MOST appropriate to be performed in order to achieve early skill acquisition? A. Distributed practice B. Blocked order C. Random order D. Serial order

B. Blocked order

A patient presents with partial-and full-thickness burns on the chest regions. The PT decides to apply TENS before debridement to modulate pain. Which TENS mode should provide the BEST relief? A. Acupuncture-like (low-rate) TENS. B. Brief intense TENS. C. Noxious TENS. D. Conventional (high-rate) TENS.

B. Brief intense TENS.

A physical therapist is treating a patient with presence of geotropic nystagmus on head turns in the supine position. The nystagmus was weaker on the right side. Which of the following is the MOST APPROPRIATE sequence of administering the intervention for these symptoms? A. Canalith repositioning maneuver with head in 20 deg extension, head turn to 90 deg to right, then moved to the left, then get the patient in prone position with neck in flexion and finally sit the patient up. B. Canalith repositioning maneuver with head in 20 deg flexion, head turn to 90 deg to left, then moved to the right, then get the patient in prone position with neck in flexion and finally sit the patient up. C. Canalith repositioning maneuver with head in 20 deg extension, head turn to 90 deg to left, then moved to the right, then get the patient in prone position with neck in flexion and finally sit the patient up. D. Canalith repositioning maneuver with head in 20 deg flexion, head turn to 90 deg to right, then moved to the left, then get the patient in prone position with neck in flexion and finally sit the patient up.

B. Canalith repositioning maneuver with head in 20 deg flexion, head turn to 90 deg to left, then moved to the right, then get the patient in prone position with neck in flexion and finally sit the patient up.

A physical therapist is developing an exercise program for a patient who has upper extremity lymphedema. Which of the following exercises should the patient perform FIRST? A. Elbow flexion B. Cervical rotation C. Shoulder abduction D. Shoulder circumduction

B. Cervical rotation

A 68-year-old patient is performing a Bruce protocol on an inclined treadmill. The PT is monitoring the patient using ECG leads. During the exercise, the PT sees the first-degree AV block ECG pattern shown. The PT's BEST response should be: A. Stop the treadmill session immediately and call the cardiologist B. Continue without any modifications and monitor ECG C. Reduce the treadmill speed and monitor ECG D. Stop the treadmill, have the patient rest, and then resume at a lower intensity

B. Continue without any modifications and monitor ECG

A PT is administering iontophoresis for the management of fungal infection. The conductive area is 10 cm2 and the current intensity is 10 mA. Which ion, polarity and current density should be used? A. Zinc, negative pole at 0.2 mA/cm2 B. Copper, positive pole at 1 mA/cm2 C. Iodine, negative pole at 0.5 mA/cm2 D. Copper, negative pole at 1 mA/cm2

B. Copper, positive pole at 1 mA/cm2

Which of the following burn wound classifications is MOST likely to produce the keloid scar shown in the picture? A. Superficial burn B. Deep partial-thickness burn C. Superficial partial-thickness burn D. Subdermal burn

B. Deep partial-thickness burn

A PT is reviewing the lab values of a 58-year-old male patient. Which of the following about glucose monitoring is LEAST appropriate? A. Hypoglycemia is documented for fasting blood glucose value of 40mg/dL B. Diabetes mellitus is when fasting blood glucose levels are between 80 and 110 mg/dL measured on 2 separate days C. Glycosylated Hemoglobin normal reference range is 4-6% D. A1C level above 10% requires immediate insulin therapy

B. Diabetes mellitus is when fasting blood glucose levels are between 80 and 110 mg/dL measured on 2 separate days

A 48-year-old female presents with chief complaints of urinary incontinence. The PT suspects that the patient may have overflow incontinence. A positive response to which of the following questions could have led the PT to arrive at that conclusion? A. Do you urinate more than 8 times a day? B. Do you dribble the urine during the day or night? C. Do you have premature loss of urine because you can't unfasten your pants on time? D. Do you ever lose urine when you laugh or cough suddenly?

B. Do you dribble the urine during the day or night?

A PT assesses a patient's vocal sounds as part of a respiratory examination. The therapist positions the stethoscope over the thorax and asks the patient to say "E". Which type of voice sound is assessed using this technique? A. Bronchophony B. Egophony C. Pectoriloquy D. Rhonchi

B. Egophony

The patient's medical history includes hypercholesterolemia, and type 2 diabetes. The patient's systolic BP is 122 mm Hg, and diastolic BP is 77 mm Hg. Which of the following categories MOST appropriately describes the type of hypertension? A. Normal B. Elevated C. Pre-hypertension D. Stage 1

B. Elevated

A PT is performing spirometry to assess the lung function. After a normal exhalation, the PT asks the patient to exhale the maximal amount of air that he can. Which of the following parameters is being assessed with this action? A. Inspiratory reserve volume B. Expiratory reserve volume C. Total lung capacity D. Tidal volume

B. Expiratory reserve volume

To prevent contractures in a patient with an above knee (transfemoral) amputation, emphasis should be placed on designing a positioning program that maintains range of motion in hip: A. Flexion and abduction. B. Extension and adduction. C. Adduction and lateral rotation. D. Flexion and medial rotation

B. Extension and adduction.

A patient was recently fitted with a patellar tendon-bearing (PTB) socket after a transtibial amputation. The patient comes for a follow up visit and the PT notes redness of the skin upon inspection. Redness on which of the following structures should be a concern for the PT? A. Patellar tendon B. Fibular head and neck C. Medial tibial plateau D. Tibial and fibular shaft

B. Fibular head and neck

In PT session, the therapist commands the patient to wear a shirt, but the patient is unable to complete the task due to inability to find buttons on his shirt. Which of the following is the MOST APPROPRIATE diagnosis for this deficit? A. Form discrimination B. Figure-ground discrimination C. Vertical disorientation D. Topographical disorientation

B. Figure-ground discrimination

A patient with a SCI is having difficulty learning how to transfer from mat to wheelchair. The patient just cannot seem to get the idea of how to coordinate this movement. The MOST effective use of feedback during early motor learning is to: A. Provide feedback only after a brief delay. B. Focus on knowledge of results. C. Focus on proprioceptive inputs. D. Focus on knowledge of performance

B. Focus on knowledge of results.

20-year-old female, after an uncomplicated delivery has a 3-cm diastasis recti and weak abdominal muscles. Which of the following exercises is MOST appropriate for the patient? A. Sitting on a swissball B. Head lifts with arms bracing the abdomen C. Double leg lifts D. Deep breathing exercises in bridging position

B. Head lifts with arms bracing the abdomen

Patient is an 8 weeks post op case of Achilles tendon repair. Patient has recently started discontinuing the CAM boot and unable to bear weight with complains of pain with weight bearing. The PT should FIRST emphasize: A. Stretching of posterior compartment muscles in standing B. Heel lift C. Continue with CAM boot D. Notify the doctor

B. Heel lift

A physical therapist is examining a 54-year-old patient and diagnoses a right pelvic drop during walking at self-selected walking speed. Which of the following is a potential cause of right pelvic drop? A. Weak hip abductors on right side B. Hip abductor contracture on right side C. Weak hip adductors on left side D. Hip abductors contracture on left side

B. Hip abductor contracture on right side

A 37-year-old female is admitted to the hospital with symptoms of low back pain. The PT is assessing the functioning of the hip muscles. Passive insufficiency of right iliopsoas will be caused by? A. Hip extension and right lateral flexion of the trunk B. Hip extension and left lateral flexion of the trunk C. Hip flexion and right lateral flexion of the trunk D. Hip flexion and left lateral flexion of the trunk

B. Hip extension and left lateral flexion of the trunk

A 74-year-old retired contractor walks to the treatment room. Patient presents with backward trunk lean during the swing phase of gait. What is the MOST likely cause ? A. Gluteus maximus weakness B. Hip flexor weakness C. Gluteus mediusweakness D. Vastus medialis weakness

B. Hip flexor weakness

A patient complaints of R knee discomfort while using their Bichannel Adjustable Ankle Locking (BiCAAL) device. The PT observed the R knee collapse when the foot makes contact to the ground during early stance. Which of the following is MOST likely causing this impairment? A. Adequate knee lock B. Inadequate dorsiflexion stop C. Inadequate plantarflexion stop D. Excessive dorsiflexion stop

B. Inadequate dorsiflexion stop

The PT is evaluating a patient with spinal cord injury. Which of the following is the MOST reasonable expectation of a patient with a C7 level spinal cord injury with respect to transfers? A. Completely unable to perform transfers on even surfaces B. Independent with transfers on even surfaces C. Independent with floor to wheelchair transfer D. Completely unable to perform transfers on uneven surfaces

B. Independent with transfers on even surfaces

A PT is treating a 6-year-old patient. The patient is seen to be standing with a toe-in posture. Which of the following postural strategies most accurately correlates with the observed foot position? A. Internal tibial torsion, increased femoral retroversion B. Internal tibial torsion, increased femoral anteversion C. External tibial torsion, increased femoral retroversion D. External tibial torsion, increased femoral anteversion

B. Internal tibial torsion, increased femoral anteversion

A patient presents with inability to sense and name the alphabets that the PT is drawing on the patient's hand. Which of the following is MOST likely to be affected along with this presentation? A. Temperature sensation B. Joint proprioception C.Painsensation D. Crude touch sensation

B. Joint proprioception

During assessment of skin sensation, which of the following structures are responsible for transmission of the cold sensation? A. Meissner corpuscles B. Krause end bulbs C. Golgi tendon organs D. Ruffini endings

B. Krause end bulbs

A PT examination reveals posterior superior iliac spine (PSIS) is low on the left; anterior superior iliac spine (ASIS) is high on the left. Interventions should MOST likely include unilateral isometric contraction of the: A. Left hip flexors to correct left anterior rotated innominate B. Left hip flexors to correct left posterior rotated innominate C. Left hip abductors to correct pelvis upslip D. Right hip adductors to correct iliac inflare on the left

B. Left hip flexors to correct left posterior rotated innominate

A 31-year-old pregnant obese female, in her third trimester, is diagnosed with damage of pudendal nerve. The patient is referred to PT clinic for pelvic floor strengthening. What is the BEST initial physical therapy intervention? A. Supine and perform a maximum of 5 contractions held for 3 seconds each B. Left side lying and perform a maximum of 10 contractions held for 5 seconds each C. Right side lying and perform a maximum of 10 contractions held for 5 seconds each D. Sitting and perform a maximum of 10 contractions held for 10 seconds each

B. Left side lying and perform a maximum of 10 contractions held for 5 seconds each

A 76-year-old female presents with diagnosis of secondary lymphedema. The patient states that she had breast cancer last year and had surgery to remove the cancer. Which of the following causes are NOT a cause/form of secondary lymphedema? A. Infection B. Milroy's disease C. Fibrosis D. Chronic venous insufficiency

B. Milroy's disease

A patient suffered a traumatic brain injury from a fall out of bed resulting in a subdural hematoma. When intervening the patient, the clinician finds the patient opens their eyes to speech, and they make conversation but use inappropriate words. Upon examination, the patient withdrawals from touch and rolls their eyes. What level of severity would the patient MOST LIKELY be classified as? A. Mild B. Moderate C. Severe D. Very Severe

B. Moderate Eye opening: 3 Motor response: 4 Verbal response: 3 TOTAL: 10

A 6-year-old male child is referred to PT. The mother is concerned that child is using his hands to climb up their legs in order to stand up from the ground. Which of the following condition the PT can suspect? A. Cerebral Palsy B. Muscular Dystrophy C. Autism Spectrum Disorder D. Down Syndrome

B. Muscular Dystrophy

A patient reports fatigue, proximal upper extremity weakness, and double vision that increases in intensity as the day progresses. The patient demonstrates bilateral ptosis of the eyelids, difficulty chewing, dysphagia, and inability to raise the eyebrows. Which of the following conditions is MOST likely present? A. Bell palsy B. Myasthenia gravis C. Trigeminal neuralgia D. Amyotrophic lateral sclerosis

B. Myasthenia gravis

The 54-year-old male patient's chart states that they have been taking beta-blockers for the past 5 years. Prior to starting an exercise training program, the patient should receive an explanation of the: A. Greater benefits from cardiovascular exercise to be achieved at lower SBP rather than at higher SBP levels. B. Need to use measures other than heart rate to determine intensity of exercise. C. Greater benefits from cardiovascular exercise to be achieved at lower HR than at higher HR levels. D. Need for longer warm-up periods and cool-down periods during exercise sessions.

B. Need to use measures other than heart rate to determine intensity of exercise.

A patient diagnosed with congestive heart failure is participating in cardiac rehabilitation. During treadmill training the patient becomes fatigued with an increased pulse and respiratory rate. In which situation would the clinician terminate the exercises? A. Having an S1 and S2 heart sound B. Onset of diaphoresis C. An increased respiratory rate of 36 breaths per minute D. A decrease in diastolic blood pressure of 8 mmHg

B. Onset of diaphoresis

A patient recovering from a traumatic brain injury seems disoriented and unable to correctly state the time of the day and his location. Which of the following is the MOST appropriate action for the PT to take? A. Immediately notify the patient's neurologist on call B. Orient the patient to the correct time and location. C. Tell caregiver to give patient a low dose of medicine to help with recalls D. Document dementia in the patient's chart

B. Orient the patient to the correct time and location.

A 45-year-old male presents to the clinic with ankle pain. Which of the following finding indicates that the patient needs to be referred out for imaging? A. Reduced ROM of ankle dorsiflexion B. Pain on palpation over tip of lateral malleoli C. Reduced strength of ankle dorsiflexors D. Pain on palpation over the talus

B. Pain on palpation over tip of lateral malleoli

A 32-year-old male patient is referred to PT for gait training post right tibial plateau fracture. The patient is currently on non-weight bearing precautions for the right lower extremity. Which of the following is the MOST appropriate to be used to train this patient? A. Bilateral canes B. Parallel bar C. Hemiwalker D. Single crutch

B. Parallel bar

A patient underwent surgical repair of Full Thickness Rotator cuff tear 6 days ago. PT wants to perform passive and assisted movement. Which position will be MOST appropriate for the patient? A. Patient in sitting with arm abducted 70 degrees with slight flexion. B. Patient is supine with arm abducted 45 degrees with slight flexion. C. Patient in sitting with arm abducted 15 degrees with slight flexion. D. Patient in supine with arm abducted 90 degrees with slight flexion.

B. Patient is supine with arm abducted 45 degrees with slight flexion.

A PT administers a series of cranial nerve tests to a patient with a confirmed lower motor neuron disease. On assessment, the patient presents as shown in the video. Which is the MOST appropriate conclusion based on this assessment? A. L hypoglossal nerve lesion B. R hypoglossal nerve lesion C. L glossopharyngeal nerve lesion D. R glossopharyngeal nerve lesion

B. R hypoglossal nerve lesion

Three DPT students are looking at effects of drugs on falls. The students are using data from previous patient medical records to compare drug usage in fallers and non-fallers. Which of the following will be the MOST appropriate study design? A. Clinical case report B. Retrospective cohort C. Randomized controlled trials D. Prospective cohort

B. Retrospective cohort

A patient presents with neurological symptoms so the PT decides to do cranial nerve assessment. When PT asks the patient to say "Ahh", the PT finds the patient's uvula deviated . Which cranial nerve is MOST likely affected? A. Left vagus nerve B. Right vagus nerve C. Left hypoglossal nerve D. Right glossopharyngeal nerve

B. Right vagus nerve

A patient with congestive heart failure is undergoing cardiac rehabilitation and is on Digoxin since the last few weeks. While monitoring the patient for signs and symptoms of digitalis toxicity, which of the following is NOT a common sign of digitalis toxicity? A. Disorientation and delirium B. Ringing in the ears C. Fatigue and Palpitations D. Gastrointestinal disturbances

B. Ringing in the ears

A 21-year-old male works as a cashier at a gas station. He is having trouble giving cash back to a customer due to lack of pronation range of motion. PT should do the following to the radial head at proximal radioulnar joint: A. Roll and slide posteriorly B. Roll anteriorly and slide posteriorly C. Roll and slide anteriorly D. Roll posteriorly and slide anteriorly

B. Roll anteriorly and slide posteriorly

78-year-old patient was admitted with a diagnosis of acute exacerbation of COPD. The patient has a FEV1 of 40% with FEV1/FVC <70% and has shortness of breathing during ambulation. According to the GOLD classification, the patient would MOST likely be classified as: A. Very severe B. Severe C. Moderate D. Mild

B. Severe

A PT examines the gait of a 62-year-old male patient. The patient exhibits right early heel off during stance phase of gait as shown in the picture. Which of the following impairments is MOST likely associated with this finding? A. Shortening of the hamstrings B. Shortening of the gastrocnemius C. Weakness of the tibialisanterior D. Weakness of the iliopsoas

B. Shortening of the gastrocnemius

A 44-year-old male patient is being evaluated by a physical therapist. The patient underwent a hernia repair 3 weeks ago and should AVOID which of the following activities? A. Walking at a metabolic equivalent of 4 after 3weeks after surgery B. Stretching of the anterior spinal and hip musculature before the incision is fully healed C.Stretching of the posterior spinal and hip musculature before the incision is fully healed D. Wall sits performed in an upright position

B. Stretching of the anterior spinal and hip musculature before the incision is fully healed

A 32-year-old healthy male is working out on a stationary bike in an outpatient physical therapy clinic. After the first 4 minutes of constant-load, sub-maximal exercise, VO2 reaches steady state, indicating that: A. Levels of lactic acid in the blood has reached steady state. B. The ATP demand is being met aerobically. C. The exercise should be discontinued immediately. D. The respiratory rate is insufficient to meet the ATP demand.

B. The ATP demand is being met aerobically.

A PT is treating a patient with Epley maneuver starting with the head in 30 degrees extension and right ear towards the ceiling. Which of the following test is MOST likely expected to be positive for the therapist to choose this intervention? A. Torsional up beating nystagmus with right dix hall pike test B. Torsional up beating nystagmus with left dix hall pike test C. Geotropic nystagmus on supine roll test D. A geotropic nystagmus on supine roll test

B. Torsional up beating nystagmus with left dix hall pike test

A PT is treating a patient with Epley maneuver starting with the head in 30 degrees extension and head turned to left. Which of the following test is MOST likely expected to be positive for the therapist to choose this intervention? A. Abnormal smooth pursuits B. Torsional up beating nystagmus with left dix hallpike test C. Geotropic nystagmus on supine roll test which is stronger on left D. Positive head thrust test to left

B. Torsional up beating nystagmus with left dix hallpike test

A patient is complaining of left lower abdominal pain. The patient reports low back pain accompanied with weight loss, nausea, vomiting, and has had bloody stool lately. The patient's pain is MOST likely due to which diagnosis? A. Chron's disease B. Ulcerative colitis C.Appendicitis D.Acute pancreatitis

B. Ulcerative colitis

A patient presents to the clinic with history of burns on the left hip and thigh region causing tight hip flexors. What gait abnormality is MOST likely to be demonstrated by the patient? A.A shorter step length with the left lower extremity B.A shorter step length with the right lower extremity C.Backward lean during stance phase on left lower extremity D.Lateral lean during swing phase on right lower extremity

B.A shorter step length with the right lower extremity

A therapist is treating a 6-year-old child with autism in outpatient pediatric physical therapy. Which of the following is the MOST appropriate intervention? A.Tumbling on mat to promote core strength. B.Ball toss, hopping, and running with a partner with verbal cues for each task C.Perform activities in a fast-paced environment with long explanations for sequencing D.Performing standing dance routine in different rooms to improve generalizability of a skill

B.Ball toss, hopping, and running with a partner with verbal cues for each task

A patient presents to the clinic for chronic low back pain. He reports the pain increases on standing and on bending to lift, sitting helps to alleviate the pain. On assessment PT found negative slump and SLR tests. Which of the following is the MOST appropriate intervention based on patient's presentation? A.Bed rest for 2 weeks along with hot packs 3 times/day for 15 minutes each B.Flexion biased exercises C.High velocity thrust to the lumbar spine to improve lumbar extension D. Extension biased exercises

B.Flexion biased exercises

A 34-year-old pregnant female is performing pelvic floor exercises in the supine position. She reports dizziness, sweating and nausea. Which condition is MOST likely present based on the patient position and diagnosis? A.Increased pressure on the inferior vena cava causing hypertensive syndrome. B.Increased pressure on the inferior vena cava causing hypotensive syndrome. C.Increased pressure on the superior vena cava causing hypotensive syndrome. D.Increased pressure on the superior vena cava causing hypertensive syndrome.

B.Increased pressure on the inferior vena cava causing hypotensive syndrome.

A 19 year old female patient is being seen in a PT clinic for pain which has worsened during the swimming season. She complains of dullness and heaviness in the shoulder, and pain with prolonged activity. She tests 7/9 on a Beighton score test. What is the best intervention for her initial treatment session? A.Modified planks and side planks B.Light multidirectional perturbations of the arm held at 90 degrees flexion in supine C.Inferior and posterior grade III mobilizations of the glenohumeral joint D.Moist heat to the shoulder at the beginning of the session for pain modulation

B.Light multidirectional perturbations of the arm held at 90 degrees flexion in supine

A 45-year-old female presents to PT with a recent diagnosis of MS. Upon initial evaluation, she complains of eye pain, vision loss in one eye and visual field loss. Which of the following cranial nerves listed below is most likely involved? A.Facial (VII) B.Optic (II) C.Abducens (VI) D.Oculomotor (III)

B.Optic (II)

A PT instructed a patient in home postural drainage positioning. Which of the following position will be MOST appropriate for right lateral segment? A.Position A B.Position B C.Position C D.Position D

B.Position B

The increased metabolic demand placed on the heart during exercise can be best estimated by examining the A.Systolic blood pressure B.Rate product pressure C.Diastolic blood pressure D.Heart rate

B.Rate product pressure

A 21-year female player has limited ROM and pain on the right shoulder. The PT plans an intervention program, but the patient refuses any treatment. She agrees that without intervention the symptoms might worsen. Which is MOST appropriate PT response? A.Call the coach of the team and report that the injury might affect the players performance B.Respect the player's decision to not use physical therapy services C.Start the treatment as the patient will most likely join in once she sees the benefits of the treatment. D.Refer the patient to another therapist who is a sports specialist

B.Respect the player's decision to not use physical therapy services

Following surgery of the right hip, a patient ambulates as shown in the picture. As part of the intervention, the PT decides to include FES to help improve the gait pattern. Stimulation should be initiated for: A.Right abductors during swing on the right. B.Right abductors during stance on the right. C.Left abductors during stance on the left. D.Left abductors during swing on the left.

B.Right abductors during stance on the right.

A 35-year-old female presents with swelling in the lower extremities. Upon examination, the PT palpates skin surfaces that are uneven with indentations and knotted fat structures. Additional findings are fat build ups around the pelvis, buttocks, hip, and a negative stemmer's sign. What is the patient's diagnosis? A.STAGE 3 lipedema B.STAGE 2 lipedema C.STAGE 1 lymphedema D.STAGE 4 lipedema

B.STAGE 2 lipedema

A young adult underwent right Achilles tendon repair 6 weeks ago and is now able to fully weight bear. The PT is giving him advice on proper shoe modification. Which of the following would be BEST for the patient to utilize? A.Normal shoes B.Shoes with 1-1.5 cm heel lift C.Shoes with lower than the regular heel D.Shoes with 5 cm heel lift

B.Shoes with 1-1.5 cm heel lift

A PT needs to check the walking gait of patient who suffered from CVA and decides to evaluate muscle function. The 3 hamstring muscles contract eccentrically during which phase of gait cycle? A.Initial Swing B.Terminal Swing C.Midstance D.Terminal Stance

B.Terminal Swing

The therapist is putting pressure on the patient's head. Which of the following patient responses will help the PT identify that the pain is nonorganic ? A.The patient reports fear of falling as the therapist increases the pressure B.The patient reports severe back pain C.The patient reports severe pain on the forehead D.The patient showed turns the head to the right side

B.The patient reports severe back pain

A patient has been admitted to the inpatient unit of a big hospital system for traumatic brain injury. The patient has been classified as RLA stage II. On auscultation, the patient is found to have decreased breath sounds and mild crepitus. Which of the following is MOST appropriate intervention in this case? A.Head low position for postural drainage and improved ventilation B.Turning the patient in bed every few hours C.Teaching the patient assisted coughing technique D.Teaching the patient to do active cycle of breathing technique

B.Turning the patient in bed every few hours

How might operant conditioning be used to shape behavior in a noncompliant pt with a TBI?

Behavior modification uses operant conditioning techniques by ID the desired behavior and reinforcing the desired behavior immediately in a meaningful way. Negative behaviors are ignored, aversive behavior is minimized, and a closed environment with few distractions is preferred

A 60 years old male is undergoing cardiac rehabilitation post MI in the hospital. On Day 3, patient has progressed to walking. Which of the following is MOST appropriate statement to document the activity ? A. Patient ambulated for 20 minutes with RPE 10/20 B. Patient ambulated 300 ft with RPE 4/20 C. Patient ambulated 500 ft in 6 minutes with RPE 10/20 D. Patient ambulated in the ICU hallway with RPE 10/20

C .Patient ambulated 500 ft in 6 minutes with RPE 10/20

A 60-year-old male is undergoing cardiac rehabilitation post complicated MI in the hospital. On day 2, the PT wants to progress the patient to sitting. Which of the following is an appropriate INITIAL task for this patient? A Make the patient stand and do weight bearing activities. B Make the patient sit on the upright chair during the visitor's time C Make the patient sit on reclining chair and check vitals D Patient is not ready for upright posture yet.

C Make the patient sit on reclining chair and check vitals

A 47-year-old mine-worker was admitted to acute care for burn injury. The PT observes that the anterior surface of the patient's trunk and both anterior lower extremities were affected. Which of the following BEST represents the percentage of body surface area involved? A. 18% B. 27% C. 36% D. 45%

C. 36%

A PT student complains low back pain due to excessive studying during NPTE week and is using hot packs in the PT clinic. When using hot packs, which of the following scenarios is MOST likely to result in burns? A. Using 6 layers of toweling between the hot pack and the patient B. A treatment time of 30 minutes C. A hot pack heated by immersion in water heated to 205 degrees Fahrenheit D. The patient's skin having too many hair

C. A hot pack heated by immersion in water heated to 205 degrees Fahrenheit

A PT is educating one of their patients regarding the effects of pregnancy and the implication for positioning and posture. Which of the following is the LEAST appropriate recommendation by the therapist? A. Patient with pregnancy induced hypertension must not do high intensity exercises B. Breath-holding exercising, and Valsalva's maneuver should be avoided C. A small-wedge must be used under the left hip while sleeping in supine in the second semester D. Borg's scale score of 12-14 during exercise is acceptable for uncomplicated pregnancy

C. A small-wedge must be used under the left hip while sleeping in supine in the second semester

A patient presents with sharp pain at the ankle. According to the patient, the ankle was injured while playing basketball last week. The PT wants to rule out any major ankle injuries. By performing this test as shown in the video (Thompson), what condition is the PT ruling out? A. Medial Meniscus tear B. Anterior talofibular ligament injury C. Achilles tendon rupture D. Calcaneofibular ligament tear

C. Achilles tendon rupture

A patient with a long history of cigarette smoking has been admitted to the hospital and presents with signs of infection, tachycardia, abnormal breath sounds in both lower lobes and dullness to percussion. The PTs major focus with this patient is: A. Getting the patient to quit smoking B. Breathing reeducation to increase efficiency of ventilation C. Airway clearance and secretion removal D. Graded inspiratory muscle training

C. Airway clearance and secretion removal

A 28-year-old female patient with AKA presents to a PT clinic. The patient must maintain the knee in extension while weight-bearing. Where should the ground reaction force line fall in order to maintain the knee in extension? A. Posterior to the axis of the right knee joint B. Lateral to the axis of the right knee joint C. Anterior to the axis of the right knee joint D. Medial to the axis of the right knee joint

C. Anterior to the axis of the right knee joint

A PT is ambulating a 75-year-old male who is in recovering from a prostatectomy procedure. The patient complains of palpitations, shortness of breath and fatigue. What should be the interpretation and immediate action according on the ECG strip shown below? A. Ventricular fibrillation; Call for a defibrillator B. Premature ventricular contractions; Take him to his bed and monitor for changes in ECG. C. Atrial Fibrillation; Stop exercise and report to the physician. D. 3rd degree heart block; Activate emergency.

C. Atrial Fibrillation; Stop exercise and report to the physician.

A 57-year-old male patient is having difficulty with walking. The PT documents the knee extension strength to be 2-/5 and incorporates EMG biofeedback to help increase motor recruitment. Which of the following protocol is the MOST appropriate to increase motor recruitment for the knee extensors? A. Begin with electrodes close together and biofeedback sensitivity is low B. Begin with electrodes widely spaced and biofeedback sensitivity is low C. Begin with electrodes widely spaced and biofeedback sensitivity is high D. Begin with electrodes close together and biofeedback sensitivity is high

C. Begin with electrodes widely spaced and biofeedback sensitivity is high

A 25-year-old male patient is taking B blockers for 3 years post Myocardial Infarction. Which of the following is the MOST likely response to exercise seen in this patient ? A. Rapid increase in HR B. Decreased exercise tolerance C. Blunted response of HR with exercise D. Increased BP

C. Blunted response of HR with exercise

A PT is treating a patient with traumatic brain injury. The patient is classified as confused and agitated per the RLA classification. The patient also has memory impairments and is unable to recall the day/location or the activities performed in the previous sessions. Which of the following is the LEAST appropriate recommendation for the physical therapist? A. Establish a daily routine for the patient B. PT should orient the patient place and time frequently C. Challenge the patient to provide the location/date and PT should not give that information D. Use chart/white board in the patient's room to document progress for the patient

C. Challenge the patient to provide the location/date and PT should not give that information

A patient presents to clinic with a posterior herniated nucleus pulposus at L3-L4. During the traction treatment in supine, the patient reports increased localized back pain. Which of the following actions is MOST appropriate for the PT to take? A. Change the position of the patient to side lying B. Refer the patient back to the physician C. Change the position of the patient to prone D. Reduce the traction force to 15% of body weight

C. Change the position of the patient to prone

A 76-year-old patient has been transferred to an acute care unit with symptoms of CHF. During auscultation, the PT finds S3 heart sound. Which of the following high-pitched breath sound is MOST likely associated with the diagnosis of CHF? A. Rhonchi B. Wheeze C. Crackles D. Pleural rub

C. Crackles

A male patient complains of having significant weight gain in abdomen and in the face. The labs show high levels of cortisol and elevated blood glucose and high levels of ACTH coming from pituitary gland. The patient MOST LIKELY has this diagnosis: A. Addison's disease B. Hashimoto's disease C. Cushing's disease D. Hypothyroidism

C. Cushing's disease

A 45-year-old female patient is being evaluated in an outpatient clinic by a physical therapist. The PT performs the test shown in the picture. A positive result of the test likely indicates the presence of which of the following conditions? A. Radial collateral ligament tear B. Carpal tunnel syndrome C. De Quervain disease D. Restricted ulnar deviation

C. De Quervain disease

A physical therapist is evaluating the specificity of the Phalen test in a group of computer programmers. Which of the following results indicates a true negative finding? A. Patients with a history of wrist pain and carpal tunnel syndrome will test negatively. B. Patients with a history of wrist pain and carpal tunnel syndrome will test positively. C. Patients with no history of wrist pain and carpal tunnel syndrome will test negatively. D. Patients with no history of wrist pain and carpal tunnel syndrome will test positively.

C. Patients with no history of wrist pain and carpal tunnel syndrome will test negatively.

A researcher is collecting data on effect of muscle stretching on passive hip range of motion in 20 patients. The researcher will provide stretching every day for a week and measure ROM every day. Which of the following experimental designs is the MOST appropriate for this type of study? A. Divide the patients into 2 groups with 10 subjects in experimental and 10 in control group, measure ROM on day 7. B. Divide the patients into 2 groups with 3 subjects in experimental and 17 in control group, measure ROM on day 1 and 7. C. Divide the patients into 2 groups with 10 subjects in experimental and 10 in control group, measure ROM on both days 1 and 7. D. Divide the patients into 2 groups with 10 subjects in experimental and 10 in control group. For the experimental group, measure ROM on day 7. For the control group, measure ROM only on day 1.

C. Divide the patients into 2 groups with 10 subjects in experimental and 10 in control group, measure ROM on both days 1 and 7.

On assessing the gait of a patient, the PT notes that the patient has trunk lean to the left in the left stance phase of the gait cycle. Which of the following should the PT check in order to determine the potential cause of this finding? A. Examine left quadriceps B. Examine right gluteus maximus C. Examine left gluteus medius D. Examine right abdominal muscles

C. Examine left gluteus medius

A newborn is examined at birth using the APGAR screening test. The following observations are made at 5 min: heart rate 120bpm, strong crying, grimace with irritability testing, kicking and arm movement are present, and the baby is pink in color throughout the body. The PT should: A. Monitor the APGAR score again at 10 minutes B. Expect respiratory complications due to low score on respiration C. Expect good heart rate, good respiration and normal muscle tone D. Expect neurological complications due to low score on reflex irritability

C. Expect good heart rate, good respiration and normal muscle tone

A patient has been referred for chronic pain. The patient complains of fatigue and decreased range of motion which worsens with stress. Upon examination PT finds multiple tender points but no referred pattern of pain or no muscle tightness. Which diagnosis is MOST LIKELY? A. Lumbar radiculopathy B. Myofascial pain syndrome C. Fibromyalgia D. Osteoarthritis

C. Fibromyalgia

A physical therapist is evaluating a client in who is referred post-MVA with a diagnosis of truncal cerebellar ataxia. She is presenting with impaired coordination, balance, gait as well as some motor coordination symptoms on her right side, affecting her ability to ambulate and complete activities of daily living independently. Which test can therapist perform on the client to examine ataxia in upper extremities? A. Smooth pursuit test B. Pendular test C. Finger to nose test D. Rapidly alternating movement test

C. Finger to nose test

A patient has been diagnosed with a decompensated congestive heart failure. Which of the following sign/symptom is LEAST likely expected to be seen in this case? A.Presence of pulmonary edema on chest X-ray B.Presence of indentations when pressure is applied to pretibial area C. Gradual decrease in weight of the patient D. Presence of low frequency heart sound in early diastole

C. Gradual decrease in weight of the patient

A PT examines a patient complaining of tingling into the 4th and 5th digits with muscle wasting over the hypothenar eminence. The PT suspects ulnar neuropathy and decides to examine the integrity of the nerve. Which of the following testing procedure would be the BEST? A. Have the patient flex both wrists while holding them for one minute B. Have the patient make a fist around the thumb and perform ulnar deviation C. Have the patient grasp a piece of paper between their first and second fingers while the examiner pulls the paper and monitors the first finger D. Have the patient perform extension of the third digit of the hand against PTresistance

C. Have the patient grasp a piece of paper between their first and second fingers while the examiner pulls the paper and monitors the first finger

A PT is instructing a patient to move the head horizontally while maintaining focus on a stable target. Which of the following tests is MOST likely to be positive for which the therapist may have recommended this test? A. Dix Hallpike test B. Finger-nose test C. Head Thrust test D. Supine roll test

C. Head Thrust test

A patient is in the emergency room with complains of lower extremity cramping. The patient had the following lab values: potassium = 2.0mEq/L, BUN = 12, creatinine = 1.0mg/dL, glucose = 97mg/dL, hemoglobin = 14g/dL. What condition would the PT LIKELY observe in the patient's medical history? A. Emphysema B. Diabetes C. Heart arrhythmias D. Anemia

C. Heart arrhythmias

A 32-year-old with R AKA presents to the clinic. On evaluating the patient, the PT notices that the patient has a right trunk lean during standing. What could be the MOST likely cause? A. High lateral wall B. High anterior wall C. High medial wall D. High posterior wall

C. High medial wall

Active insufficiency of right iliopsoas will be caused by? A. Hip extension and right lateral flexion of the trunk B. Hip extension and left lateral flexion of the trunk C. Hip flexion and right lateral flexion of the trunk D. Hip flexion and left lateral flexion of the trunk

C. Hip flexion and right lateral flexion of the trunk

A 58-year-old patient has chronic obstructive pulmonary disease. During examination, the PT finds that a patient has a weak wet cough. Which of the following is MOST appropriate to help this patient clear secretions? A.Assisted coughing in the supine position B.Postural drainage in the side lying position C. Huffing D. Mechanical percussion

C. Huffing

A 56-year-old male complaints of low back pain with PMH of CHF, HTN. During the evaluation, the patient provided a list of medications with one of them indicated as Candesartan. The PT is educating on potential side effects from this medication as the following side effects MOST likely correlate with this drug: A. Hypokalemia B. Hypertension C. Hypotension D. Prevents the recurrence of atrial fibrillation

C. Hypotension

A patient presents with increased pain in the joints of the hands which started about 7 months ago. The patient has history of rheumatoid arthritis. Which of the following is the MOST appropriate recommendation regarding use of cryotherapy for chronic inflammation? A. Ice shoulder be applied for at least 45 mins at a stretch at least 1 hour apart. B. Ice should be applied multiple times during the day with interval of not more than 30 mins and for 40 mins each time. C. Ice should be applied for at most 20 minutes at a stretch at least 1 hour apart. D. Ice is an absolute contraindication for chronic inflammatory conditions.

C. Ice should be applied for at most 20 minutes at a stretch at least 1 hour apart.

A physical therapist observes gait of a 34-year-old male patient. The PT suspects a leg length discrepancy. Which of the following gait deviations is MOST likely seen by the therapist? A. Increased dorsiflexion of the short limb during swing and increased plantar flexion of the long limb during stance B. Decreased knee flexion and increased dorsiflexion of the long limb during stance and increased dorsiflexion of the short limb during swing C. Increased dorsiflexion with early heel rise of the long limb at heel off and increased plantar flexion of the short limb during stance D. Increased plantar flexion of the long limb at heel strike and decreased knee flexion of the short limb during heel off

C. Increased dorsiflexion with early heel rise of the long limb at heel off and increased plantar flexion of the short limb during stance

Two PTs are asked to perform a test on the same group of patients using the Berg Balance test. The results of both sets of measurements reveal differences in the PTs' scores, but not in the repeat measurements. This is indicative of a problem in: A. Concurrent validity B. Intrarater reliability C. Interrater reliability D. Construct validity

C. Interrater reliability

A patient 1 month post myocardial infarction is being seen in a clinic. When increasing the patient to 5 MET's of workload, the therapist notes that the ECG reads 1 unifocal PVC. The PT's IMMEDIATE action should be: A. Continue to exercise and increase the intensity B. Stop exercise because patient is undergoing ischemia C. Keep exercising at a lower intensity, consultation with physician is not required here. D. Stop exercise and consult with physician before starting any exercise

C. Keep exercising at a lower intensity, consultation with physician is not required here.

A patient presents complains of loss of sensation over the ulnar side of the hand. On evaluation, there is weakness intrinsic hand muscles leading to nonfunctional hand. Which of the following is the MOST LIKELY diagnosis? A. Radial nerve palsy B. Erb'spalsy C. Klumpke'spalsy D. C6 radiculopathy

C. Klumpke's palsy

A PT is evaluating an 86 years old female who sustained a right CVA due to an MCA infarction 3 weeks ago. Patient has the classical signs and symptoms of MCA CVA. During the initial evaluation, the PT performs reflex testing on the patient. Which of the following should the PT expect on assessment? A. Right side hyperreflexia and - Babinski B. Right side hyporeflexia and + Babinski C. Left side hyperreflexia and + Babinski D. Left side hyporeflexia and - Babinski

C. Left side hyperreflexia and + Babinski

A 44-year-old male who consumes excessive amounts of alcohol is referred to PT clinic for knee pain. During treatment, patient reports right-sided shoulder pain. Which of the following structures is the MOST likely source of the referred pain? A. Prostate B. Appendix C. Liver D. Kidney

C. Liver

A PT is examining a patient in an outpatient clinic and notices swelling on patient's right foot as shown in picture. Which of the following conditions can the PT suspect? A. Heart failure B. Lipidema C. Lymphedema D. Chronic Venous insufficiency

C. Lymphedema

The PT is trying to provide the appropriate wheelchair measurements. Which of the following options provide the correct measurement for seat depth? A. Measure the length from posterior buttock to posterior aspect of popliteal fossa and add 2 inches to the measurement. B. Measure the total area of both the hips and add 2 inches to the measurement. C. Measure the total area from posterior buttock to posterior of popliteal fossa and subtract 2 inches from the measurement. D. Measure the trunk length and subtract it from lower leg length and add 2 inches to the measurement.

C. Measure the total area from posterior buttock to posterior of popliteal fossa and subtract 2 inches from the measurement.

A physical therapist is facilitating an in-service on BLS by educating on the Do's and Don'ts of adult high-quality CPR. Which of the following SHOULD the rescuer do during the training? A. Include pauses in between compressions for greater than 10 seconds B. Compress chest to depth of less than 2 cm C. Minimize pauses in between compressions D. Compress chest at a rate of 123/min

C. Minimize pauses in between compressions

A PT is looking at squat activity in 20 obese and 20 normal weight females. Both groups performed 10 bilateral squats at 100-degrees of knee flexion. The PT hypothesizes that knee moments will be higher in obese. Which is the MOST appropriate test? A. Chi-Square test B. Mann Whitney U. test C. One tailed t-test D. Two tailed t-test

C. One tailed t-test

A patient presents to the clinic with hyperfunction of the gland shown in red in the figure. Which of the following signs and symptoms are MOST likely to be associated with this condition? A. Hypocalcemia and hyperphosphatemia B. Convulsions and tetany C. Osteopenia and peptic ulcers D. Weight gain and decreased appetite

C. Osteopenia and peptic ulcers

A 49-year-old patient presents to a clinic with swollen legs. The PT suspects lipedema as a diagnosis. Which of the following is NOT likely seen as patient's presentation? A. Patient is susceptible to bruising of the affected area. B. Patient would have a negative stemmer sign. C. Patient has a high likelihood of developing cellulitis. D. Patient would report pain on pressure.

C. Patient has a high likelihood of developing cellulitis.

A physical therapist is performing a Cozen'stest in a group of tennis players. Which of the following results indicates a true negative finding? A. Patients with a history of elbow pain and Lateral Epicondylitis will test negatively. B. Patients with a history of elbow pain and Lateral Epicondylitis will test positively. C. Patients with no history of elbow pain and Lateral Epicondylitis will test negatively. D. Patients with no history of elbow pain and Lateral Epicondylitis will test positively.

C. Patients with no history of elbow pain and Lateral Epicondylitis will test negatively.

A 28-year-old female has chief complaints of right ankle discomfort. Upon further examination, the PT identified limited inversion on the right side. The therapist would MOST appropriately perform this mobilization technique? A. Perform posterior glide of the talus while stabilizing the distal tibia B. Perform anterior glide of the talus while stabilizing the distal tibia C. Perform lateral glide to subtalar while stabilizing distal tibia D. Perform medial glide to subtalar while stabilizing distal tibia

C. Perform lateral glide to subtalar while stabilizing distal tibia

A 77-year patient post CVA presents with difficulty in dorsiflexing the left foot during heel strike. Which orthotic device is least restrictive and assists with dorsiflexion? A. Hinged ankle AFO with anterior stop B. Molded Solid ankle AFO C. Posterior leaf spring D. AFO with a resilient heel

C. Posterior leaf spring

A 6-year-old boy has been diagnosed with Duchenne muscular dystrophy, the MOST appropriate physical therapy goal is: A. Inhibition of abnormal muscle tone B. Preservation of strength and muscle tone. C. Prevention of contractures and determine the best method of mobility D. Facilitation of normal movement and improvement of strength.

C. Prevention of contractures and determine the best method of mobility

A 43-year-old female patient presents to a pro bono outpatient clinic for a maintenance program. Her past medical history includes multiple sclerosis (MS). The patient reports that over the past 2 years she has had significant flare ups and her functional mobility has been declining. Which subtype of MS does the patient LIKELY have? A. Primary-progressive MS B. Secondary-progressive MS C. Progressive-relapsing MS D. Relapsing-remitting MS

C. Progressive-relapsing MS

A 48-year-old patient with multiple sclerosis presents with dysmetria in both upper extremities. Which of the following interventions is the BEST choice to deal with this problem? A. 8-lb weight cuffs to wrists during activities of daily living training. B. Pool exercises using water temperatures greater than 88-degree F. C. Proprioceptive neuromuscular facilitation patterns using dynamic reversals with carefully graded resistance. D. Isokinetic training using low resistance and fast movement speeds.

C. Proprioceptive neuromuscular facilitation patterns using dynamic reversals with carefully graded resistance.

A 28-year-old male patient with a left above knee amputation is referred to an outpatient amputee clinic. During gait analysis, the PT observes a medial heel whip during heel off on the left side. Which of the following is the MOST likely cause of this deviation? A. Taut extension aid B. Inadequate medial rotation of the knee joint. C. Prosthetic knee bolt is externally rotated. D. Short prosthesis

C. Prosthetic knee bolt is externally rotated.

A 27-year-old male presents to the clinic with chief complaints of severe stiffness in his hands, especially the distal interphalangeal joints, and toes. His knees and elbows also feel stiff and achy. The stiffness tends to last for at least 45 minutes every morning and improves with movements. During assessment, the therapist notices diffuse swelling of the fingers and brownish-yellow discoloration of the nails and presence of well-defined, erythematous, silver-scaled papules on his skin. Which of the following is the MOST likely causeof his symptoms? A. Ehlers Danlos syndrome B. Rheumatoid arthritis C. Psoriatic arthritis D. Reiter's syndrome

C. Psoriatic arthritis

A student is in the autonomous stage of the NPTE preparation. Which of the following combinations is MOST appropriate in this stage? A.Block practice/summary feedback B. Block practice/continuous feedback C. Random practice/summary feedback D. Random practice/continuous feedback

C. Random practice/summary feedback

A patient is getting treated for pelvis pain and is accompanied by his wife to the PT session. While treating, the patient expressed dissatisfaction in his marital life and says he has planned to end his life. What would be the BEST PT action? A. Ask the patient to seek an appointment with a mental health practitioner B. Disagree with the patient and say positive things about living life C. Refer the patient immediately to mental health practitioner and stay there until help arrives D. Inform the patient's wife that this problem is outside the scope of physical therapy practice

C. Refer the patient immediately to mental health practitioner and stay there until help arrives

A 55-year-old obese male patient is taking Lipitor (a cholesterol lowering drug). Which of the following side effects should the physical therapist be MOST concerned about? A. Easy bruising B. Sleep disturbance C. Rhabdomyolysis D. Diarrhea

C. Rhabdomyolysis

A 58-year-old male underwent a total hip arthroplasty on the left side using a postero-lateral approach. The PT is working on a gait training program for the patient. The therapist should instruct the patient to hold the crutch or cane in the: A. Left hand to decrease activity in the left hip abductors. B. Left hand to facilitate activity in the left hip abductors. C. Right hand to decrease activity in the left hip abductors. D. Right hand to facilitate activity in the left hip abductors.

C. Right hand to decrease activity in the left hip abductors.

A patient with an above knee prosthetic limb is displaying right lateral trunk bending while ambulating. Which of the following would be the MOST likely cause of this gait abnormality? A. Right anterior wall is too high B. Right medial wall too low C. Right lateral wall too low D. Right posterior wall is too high

C. Right lateral wall too low

A therapist is assessing cranial nerve integrity in a patient. On shining light in the patient's right eye none of the pupils constrict, however on shining the light in patient's left eye both pupils constrict. Which of the following is MOST appropriate about this finding? A. Left optic nerve is affected B. Right oculomotor nerve is affected C. Right optic nerve is affected D. Left oculomotor nerve is affected

C. Right optic nerve is affected

A 39-year-old comes to an outpatient clinic complaining of hip problems. The PT notices a drop of the left hip during right midstance. The MOST LIKELY cause of this impairment can be the injury of: A. Right inferior gluteal nerve B. Right femoral nerve C. Right superior gluteal nerve D. Right obturator nerve

C. Right superior gluteal nerve

During the evaluation, the patient reports general muscle aches, pain, and soreness, along with increased frequency of dark-colored urine. The PT noticed that the patient has hyperlipidemia. Which of the following medications is LEAST likely related to this patient's clinical scenario? A.Atorvastatin (Lipitor) B.Pravastatin (Pravachol) C.Ropinirole (Requip) D.Simvastatin (Zocor)

C. Ropinirole (Requip)

A patient recently underwent a radical lymph node dissection secondary to prostate cancer. According the patient's history, what is MOST important for the physical therapist to educate the patient on? A.Routinely perform volumetric measurements of right lower extremities. B.Routinely use a home compression pump with pressures >80 mm Hg. C.Routinely monitor any changes in the fitting of their socks or shoes. D.Routinely perform circumferential measurements of both lower extremities

C. Routinely monitor any changes in the fitting of their socks or shoes.

A patient presents with complains of pain and numbness in the right arm extending to the thumb. The PT performs the special test as shown in the picture below. If the test is positive, which is the MOST appropriate intervention? A. Strengthen the right levator scapulae and sternocleidomastoid muscles. B. Perform (Posterior-Anterior) PA glide of the C6 vertebrae with right side bending and rotation. C. Stretch the right pectoralis minor and sternocleidomastoid muscles. D. Perform grade 3 manipulation of the first and second ribs.

C. Stretch the right pectoralis minor and sternocleidomastoid muscles.

A PT examination reveals posterior superior iliac spine (PSIS) is low on the left; anterior superior iliac spine (ASIS) is high on the left. Interventions should MOST likely include:A. Stretching the right hip flexors to correct right anterior rotated innominate B. Strengthening of left hip flexors to correct left posterior rotated innominate C. Stretching the left hip extensors to correct left posterior rotated innominate D. Strengthening the right hip extensors to correct right posterior rotated innominate

C. Stretching the left hip extensors to correct left posterior rotated innominate

A patient presents with limited mouth opening of 25 mm due to pain. There is no complain of clicking sound or mouth deviation when he opens his mouth. The patient is unable to completely close his mouth with teeth clenched together. What is the MOST likely diagnosis based on the patient symptoms? A. Hypomobility B. Disc displacement with reduction C. Synovitis D. Capsulitis

C. Synovitis

A 54-year-old patient is admitted to the hospital for lung congestion. The patient's daughter who works in the same hospital as a respiratory therapist, visits him in the hospital. She wants to look at her father's medical record. The PT should: A. Tell her to ask the consultant physician for permission. B. Tell her she cannot see the chart because she could misinterpret the information. C. Tell her that she must have the permission of her father before she can look at the chart. D. Give her the chart and let her read it as she may have some insights.

C. Tell her that she must have the permission of her father before she can look at the chart.

A 45-year-old man with lymphedema of the left leg is being examined by a PT. The PT determines that it is an early-stage II lymphedema and pitting scale grade is 3+. Which of the following statements is MOST likely the clinical presentation? A. The indentation produced is greater than 1 inch B. The indentation on finger pressure lasts for less than 15 seconds C. The indentation on finger pressure lasts for 20 seconds D. The indentation produced is less than 1/2 inch

C. The indentation on finger pressure lasts for 20 seconds

A 2-year-old patient is being treated for gait training in outpatient physical therapy. The patient has calluses on the dorsal aspect of the toes and is unable to perform MTP extension. Which of the following is MOST likely true? A. The patient has a persistent ATNR B. The patient has an integrated STNR C. The patient has a persistent plantar grasp reflex D. The patient has an integrated plantar grasp reflex

C. The patient has a persistent plantar grasp reflex

A patient with a recent episode of right-sided cerebral infarct presents with perceptual deficits and poor awareness of their impairments. Which of the following strategies is LEAST likely to be recommended while treating this patient? A. Using more verbal cues/commands while treating the patient B. When in supine position, L scapula should be protracted and shoulder slightly abducted C. Using visual cues while treating the patient D. When in L side lying position, left hip should be extended and knee flexed and supported by pillows

C. Using visual cues while treating the patient

A therapist is treating a patient with right parietal lobe lesion. He holds a cane vertically in front of the patient then turns it sideways horizontally and gives to the patient to turn it back to the original position. The patient takes the cane and places it at an angle and informs the therapist he has successfully completed the task. Which of the following MOST appropriate regarding this presentation? A. Depth perception disorder B. Visual agnosia C. Vertical disorientation D. Ideomotor apraxia

C. Vertical disorientation

An 85-year-old patient with chronic congestive heart failure was recently admitted to the hospital. Which of the following plans for prophylactic respiratory care is LEAST appropriate? A. Frequent turning and positioning every 2 waking hours B. Gentle percussion 2 times per day C. Vigorous chest vibrations, with the foot of the bed elevated, 4 times per day D. Gentle coughing and deep breathing exercises every 4 hours

C. Vigorous chest vibrations, with the foot of the bed elevated, 4 times per day

A 58-year-old patient recently suffered an above knee amputation and underwent surgery. A PT is examining the wound site and sees an exudate. Which of the following findings indicates an infected wound and should be reported to be physician? A. Dark red blood B. Bright red blood C. Viscous yellow exudate D. Serosanguineous exudate

C. Viscous yellow exudate

A 74-year-old contractor has a prominent forward lean when they are in stance phase on the right lower extremity. From this observation, the PT would hypothesize that the MOST LIKELY cause is: A. Weak gluteus maximus on the left side B. Weak gluteus mediuson the left side C. Weak quadriceps on the right side D. Weak hamstrings on the right side

C. Weak quadriceps on the right side

Excessive upward rotation of the left scapula is noted as the patient attempts shoulder abduction. Which of the following muscles is MOST likely to contribute to excessive scapular rotation? A. Weakness of serratus anterior B. Weakness of lower trapezius C. Weakness of rhomboid major & Minor D. Weakness of pectoralis major

C. Weakness of rhomboid major & Minor

A 35-year-old male presents with a skin condition as shown in the picture below. There are irregular areas of localized skin edema. Which of the following is the MOST likely diagnosis for this patient? A. Blisters B. Vesicles C. Wheals D. Pustules

C. Wheals

A patient has been diagnosed with uncompensated metabolic acidosis. Which of the following findings are MOST likely expected to be seen in the lab report? A. pH= 7.4, HCO3= 26mEq/L, PaCO2= 30 mm Hg B. pH= 7.3, HCO3= 26mEq/L, PaCO2= 55 mm Hg C. pH= 7.3, HCO3= 20mEq/L, PaCO2= 35 mm Hg D. pH= 7.5, HCO3= 19 mEq/L, PaCO2= 46 mm Hg

C. pH= 7.3, HCO3= 20mEq/L, PaCO2= 35 mm Hg

A PT examines the output from a single lead electrocardiogram of a patient in an inpatient clinic. The ECG strip is shown in picture below. PT should determine the heart rate of the patient as? A.110 beats per minute B.70 beats per minute C.80 beats per minute D.50 beats per minute

C.80 beats per minute

A 42-year-old female patient presents with chief complaints of asymmetrical weakness of lower and upper extremities. There is presence of muscle fasciculations and impaired speech, but her sensations seem relatively intact. Her husband mentions that she randomly starts laughing or crying for "no reason at all". Which of the following signs and symptoms should the therapist LEAST likely expect to be positive in this case? A.Presence of memory impairments B.Presence of foot slap while walking C.Cervical flexor muscle weakness D.Reduced vital capacity

C.Cervical flexor muscle weakness

A patient is seen after an uncomplicated vaginal delivery of her third child. During the treatment, the patient begins to complain of a headache and visual disturbances, and suddenly develops a seizure. Which condition is MOST likely? A.Preeclampsia B.Gestational diabetes C.Eclampsia D.Ectopic pregnancy

C.Eclampsia

A patient demonstrated painful ROM in abduction movement during range of motion examination of shoulder. Which of the following joint mobilization techniques is MOST appropriate for the patient? A.Large amplitude oscillations performed at the beginning of the range of motion in a superior direction B.Small amplitude oscillations into tissue resistance up to the limit of available motion in a superior direction C.Large amplitude oscillations within the available range of motion in an inferior direction D.Small amplitude oscillations into tissue resistance at the limit of available joint motion in an inferior direction

C.Large amplitude oscillations within the available range of motion in an inferior direction

Upon further examination, the second patient seems to respond to simple commands consistently, but responses are not purposeful. Verbalizations are inappropriate and patient tends to make up stories. The PT would MOST likely classify this patient on the LOCF as: A.Level VI B.Level IV C.Level V D.Level III

C.Level V

A 78-year-old male has a diagnosis of acute exacerbation of COPD. The medical record indicated the patient has an FEV1 of 68% with FEV1/FVC<70%. During the examination, the patient appeared to have SOB during ambulation. According to the GOLD classification, the patient would MOST likely be classified as: A.Very severe B.Severe C.Moderate D.Mild

C.Moderate

Which of the following does not increase the likelihood of a patient being diagnosed with hip OA? A.Pain with squatting B.Passive IR of 20 degrees C.Passive ER of 30 degrees D.Pain with stair ambulation

C.Passive ER of 30 degrees

A PT is performing cranial nerve testing on a 55-year-old male patient. When observing the patient's right pupillary response to light, the PT observes that there is no constriction of the right pupil but the left pupil does constrict. Which cranial nerve is the MOST likely cause of this impairment? A.Left Oculomotor nerve (CN III) B.Right Optic nerve (CN II) C.Right Oculomotor nerve (CN III) D.Left Trochlear nerve (CN IV)

C.Right Oculomotor nerve (CN III)

A patient with a right transfemoral amputation is referred to a clinic. During gait analysis, the PT observes that patient's trunk is bending laterally on the right side during right stance phase. Which of the following would be the MOST likely cause of this gait abnormality? A.Right socket is too small B.Right medial wall too low C.Right lateral wall too low D.Right prosthetic limb is too long

C.Right lateral wall too low

A 58-year-old patient is lifting a wine box using a stoop lift at a winery. Which of the following is the MOST significant factor in increasing compression forces on the spine in addition to the weight of the wine box? A.The height of the wine box from the ground B.Performing the lift with the lumbar spine in a neutral position C.The distance of the wine box from the base of the spine D.The muscle strength of the lower extremities

C.The distance of the wine box from the base of the spine

Differentiate between key basic life support components for adults, children, and infants in terms of compression rate and depth

Compression rate is same: 100-120/min Compression depth varies: adults 2 inches, children 1/3 AP depth (2 inches), infants 1/3 AP depth (1.5 inches)

A 29-year computer engineer presents to the outpatient clinic with neck pain. The PT decides to use traction in order to elongate the tissue and reduce muscle spasm. What is the MOST appropriate way to achieve this goal? A. 50 degrees of neck flexion with 10% body weight B. 10 degrees of neck extension with 30% body weight C. 20 degrees of neck extension with 10 % body weight D. 20 degrees of neck flexion with 10% body weight

D. 20 degrees of neck flexion with 10% body weight

Which of the following patients will be an ABSOLUTE contraindication for an aquatic therapy session with a physical therapist? A. A 22-old-year male athlete with an open wound covered with occlusive dressing B. A 40-old-year female with lymphedema post mastectomy with no prior experience in water C. A 55-old-year patient post stroke with a gastrostomytube D. A 50-old-year patient with respiratory disorder and vital capacity <1 liter

D. A 50-old-year patient with respiratory disorder and vital capacity <1 liter

A 58-year-old underwent total knee replacement surgery last week and is on anticoagulant therapy. Which of the following medications should be avoided by the patient? A. Diphenhydramine (Benadryl) to avoid risk of ecchymosis B. Methotrexate (Trexall) to avoid risk of hypotension C. Atenolol (Tenormin) to avoid risk of hypotension D. Acetylsalicylic acid (Aspirin) to avoid risk of ecchymosis

D. Acetylsalicylic acid (Aspirin) to avoid risk of ecchymosis

A physical therapist is performing joint mobilization on a patient in the outpatient clinic. The PT notes the electrocardiogram in the photograph. The physical therapist's INITIAL response should be to: A. Continue with joint mobilizations at same intensity B. Continue with joint mobilizations at a lower intensity C. Stop the treatment and monitor ECG for 10 minutes. D. Activate the emergency system or call 911

D. Activate the emergency system or call 911

All of the following are components of treatment for lymphedema EXCEPT: A. Decongest the trunk quadrants first before addressing the lymphedematous extremity B. Exercise should always be performed with a compression bandage C. Decongesting the proximal portions of the limb first and then working distally with the direction of flow always towards the trunk D. Affected limbs are bandaged with high -stretch compression bandages

D. Affected limbs are bandaged with high -stretch compression bandages

A physical therapist is observing a 67-year-old female patient walk, and notes a forward trunk lean with knee hyperextension. A LIKELY contributing cause for this deviation is: A. Excessive ankle dorsiflexion B. Quadriceps spasticity C. Ankle plantarflexion weakness D. Ankle plantarflexion contracture

D. Ankle plantarflexion contracture

A patient diagnosed with Parkinson's Disease exhibits a forward stooped posture. As part of the patient's care plan the PT selects a number of active exercises that promote improved posture. Which PNF pattern would be the MOST appropriate to achieve the PT's objective? A. Bilateral D1 Extension B. Bilateral D1 Flexion C. Bilateral D2 Extension D. Bilateral D2 Flexion

D. Bilateral D2 Flexion

When examining, the PT finds the patient's speech is slow and hesitant but auditory comprehensions are preserved. The patient's MRI scan image is shown in the picture. The patient is MOST LIKELY to present with symptoms of A. Global aphasia B. Non-expressive aphasia C. Wernicke's aphasia D. Broca's aphasia

D. Broca's aphasia

A patient with SCI has trouble wearing earrings and tying their hair. On assessment they were able to perform sit to stand transfers and maneuver the stairs relatively safely. The patient is a school teacher and their goals include being able to write on the whiteboard. Which of the following type of SCI is the patient MOST likely expected to have? A. Cauda equina syndrome B. Anterior cord syndrome C. Posterior cord syndrome D. Central cord syndrome

D. Central cord syndrome

A researcher performed Lachman's test for knee ligament injury and compared the outcomes to that of MRI findings. What type of validity is the researcher trying to establish? A. Content validity B. Construct validity C. Face validity D. Concurrent validity

D. Concurrent validity

A 66-year-old patient complains of loss of balance while walking on a thickly carpeted floor in the middle of the night when its completely dark. Which condition on the CTSIB would the patient present with increased body sway? A. Condition 2 B. Condition 3 C. Condition 4 D. Condition 5

D. Condition 5

A PT is examining a 46-year-old patient who underwent a radical mastectomy with axillary node removal. Which of the following signs would indicate INITIAL development of lymphedema in the patient? A. Non-pitting edema of lower leg and foot B. Shallow wound beds on the forearm C. Atrophy of the biceps muscle D. Decreased flexibility of the fingers

D. Decreased flexibility of the fingers

A physical therapist is administering iontophoresis for the management of musculoskeletal inflammation. To administer this treatment, which ion and polarity should be used? A. Zinc, positive pole. B. Acetate, negative pole. C. Copper, positive pole. D. Dexamethasone, negative pole.

D. Dexamethasone, negative pole.

Which of the following drugs would NOT be used to lower blood pressure in hypertensive patients? A. Thiazide diuretics B. ACE inhibitors C. Calcium channel blockers D. Digitalis

D. Digitalis

A physical therapist is planning to start their private PT practice and is working on making the clinic wheelchair accessible. Which of the following is NOT appropriate for a wheelchair accessible plan? A. Ramp slope: 1:12 B. Ramp width: 36 inches C. Tile or hardwood floors D. Door width should be 12 inches

D. Door width should be 12 inches

A 31-year-old female presents to a physical therapy clinic with agonizing, nauseating, exhausting, tiring and unbearable pain. What pain pattern is MOST related to the symptoms? A. Vascular B. Neurogenic C. Musculoskeletal D. Emotional

D. Emotional

All of the following are guidelines in treating patients with GERD EXCEPT A. Any intervention requiring a supine position should be scheduled before meals and avoided just after eating B. Encourage the patient to sleep on left side for nocturnal reflex C. Modification of position towards a more upright posture may be required if symptoms persists during therapy D. Encourage the patient to sleep on right side for nocturnal reflex

D. Encourage the patient to sleep on right side for nocturnal reflex

Occupational Safety and Health Administration (OSHA) is responsible for which of the following? A. Determining the how much compensation the therapist should get yearly B. Assuring minimum standard are met at the hospital to maintain accreditation and prevent negligence in patient care. C. Assuring minimum standard are met by the rehabilitation clinics to maintain accreditation and safety of patients. D. Ensuring adequate steps taken to prevent exposure to harmful radiation to employees at an X-ray center.

D. Ensuring adequate steps taken to prevent exposure to harmful radiation to employees at an X-ray center.

A patient with dyspnea on exertion and has an FEV1 /FVC ratio of 65%. They report chest tightness and frequent coughing. Which lung volume would be decreased compared to a patient with a healthy pulmonary system? A. Functional residual capacity B. Residual volume C. Total lung capacity D. Expiratory reserve volume

D. Expiratory reserve volume

A 6-month-old child was referred to physical therapy for right torticollis. The MOST effective method to stretch the muscle is by positioning the head and neck into: A. Flexion, left side-bending, and left rotation. B. Extension, right side-bending, and left rotation. C. Flexion, right side-bending, and left rotation. D. Extension, left side-bending, and right rotation.

D. Extension, left side-bending, and right rotation.

A male patient with BMI of 38 kg/m2 presented to an outpatient clinic with excessive fatigue. The PT observes ichthyosis on the lower extremities as shown in the picture. What would be the MOST likely cause in this patient? A. Addison's disease B. Graves' disease C. Cushing's syndrome D. Hashimoto's disease

D. Hashimoto's disease

A chronic smoker is diagnosed with COPD and undergoes ABG analysis. The physical therapist is MOST likely to see following changes in the ABG report? A. Decreased PaCO2, increased PaO2, and decreased pH. B. Decreased PaCO2, decreased PaO2, and increased pH. C. Increased PaCO2, increased PaO2, and increased pH. D. Increased PaCO2, decreased PaO2, and decreased pH.

D. Increased PaCO2, decreased PaO2, and decreased pH.

During intervention for a patient with weak tibialisanterior muscles, the PT decides to use FES to improve ambulation. Stimulation should be initiated for the weak muscles during which phase of the gait cycle? A. Mid stance to terminal stance B. Initial contact to mid stance C. Loading response to mid stance D. Initial swing to mid swing

D. Initial swing to mid swing

A physical therapist is administering iontophoresis for the management of a sclerotic scar in a patient with adhesive capsulitis. To administer this treatment, which ion and polarity should be used? A. Zinc, negative pole. B. Acetate, positive pole. C. Copper, positive pole. D. Iodine, negative pole.

D. Iodine, negative pole.

Which of the following is NOT correct regarding Coumadin? A. It should not be given to a patient with an active GI ulcer B. Adverse effects of Coumadin can be Hematuria and bleeding gums C. It inhibits the formation of Thrombin and fibrin in the clotting process D. It should be given to a patient undergoing a surgery for Liver transplant.

D. It should be given to a patient undergoing a surgery for Liver transplant.

A patient presents to the clinic with complains of knee pain. On evaluation the PT notices that the patient has excessive ankle plantarflexion along with excessive anterior pelvic tilt. What could be the correlated motion at the knee joint? A. Genu valgum B. Coxa vara C. Medial tibial torsion D. Knee hyperextension

D. Knee hyperextension

A patient with a transfemoral amputation is unable to wear a total contact prosthesis for the past 4 days. The patient reports shooting pain at the end of the residual limb. Examination of the residual limb does not show any erythema. The MOST LIKELY cause is: A. Cellulitis B. Dermatitis C. Impetigo D. Neuroma

D. Neuroma

A 50-year-old male patient is diagnosed with a second-degree heart block type II. What would a physical therapist expect to find on the ECG strip? A. An increase in PR interval lengths with no dropped beats B. No relationship between P waves and QRS complexes C. A gradual increase in PR interval length in all the beats preceding a dropped beat D. Normal PR intervals in all the beats preceding a dropped beat

D. Normal PR intervals in all the beats preceding a dropped beat

A 52-year female comes with chief complains in the R groin and upper thigh region which was gradual in onset. She has a medical history of Ulcerative Colitis for which she has been taking Prednisolone. Which of the following is MOST LIKELY diagnosis for this patient? A. Meralgia paresthetica B. Trochanteric bursitis C. Osteomyelitis of hip D. Osteoporosis of hip

D. Osteoporosis of hip

After completing an examination of a patient with shoulder pain, the PT concludes that the cause is subscapularis tendinitis. The clinical finding supportive of this conclusion is: A. Tenderness at the greater tubercle of the humerus B. Painful resisted shoulder adduction C. Pain provoked with active glenohumeral external rotation D. Pain provoked with passive glenohumeral external rotation

D. Pain provoked with passive glenohumeral external rotation

A PT is observing a client at their workstation in an office. The client's chief complaint is low back pain. The following are findings from the ergonomic assessment; forward trunk lean to get closer to screen, sitting for long periods of time with no break, no lumbar support from chair, keyboard and mouse are on an ergonomic adjustable tray, and feet are flat on the floor. What changes would the PT make to alleviate client's low back pain? A. No changes are necessary; client has adequate posture. The client must be faking their low back pain. B. Place keyboard and mouse on desktop for easier access, have client recline chair, and slouch shoulders to relax back extensors. C. Patient education to take multiple breaks throughout the day, keep the same chair, and move the monitor farther away from the client. D. Patient education to take multiple breaks throughout the day, obtain a chair with proper lumbar support, and move monitor closer to client.

D. Patient education to take multiple breaks throughout the day, obtain a chair with proper lumbar support, and move monitor closer to client.

A PT is assessing a patient with chief concerns of dizziness and loss of balance. Presence of which of the following findings during assessment will MOST likely lead the therapist to refer the patient to a physician? A. Mild Ataxia B. Positional testing reproducing the nystagmus C. Vertigo suppressed by visual fixation D. Pendular nystagmus

D. Pendular nystagmus

A 58-year-old patient comes to PT clinic for a gait and posture evaluation. The patient has a right sided transtibial amputation and is unable to flex the right knee in stance phase. What is the MOST likely cause for this gait deviation is: A. Anterior displacement of the socket relative to the foot. B. The knee bolt is rotated externally C. The prosthesis is too light D. Posterior displacement of the socket relative to the foot.

D. Posterior displacement of the socket relative to the foot.

A patient who is 3 months post right ankle fracture has an active range of motion of 0°to 5°of dorsiflexion and 0°to 20°of plantar flexion. To restore motion required for normal dorsiflexion, which of the following joint mobilization techniques the PT should perform? A. Anterior glide of the talus B. Lateral glide of the calcaneus C. Medial glide of the calcaneus D. Posterior glide of the talus

D. Posterior glide of the talus

A PT is testing the active shoulder range of motion of a 45-year-old female patient. The PT asks the patient to move the shoulder to full medial/internal rotation. During medial rotation at the shoulder joint (GH joint), the humerus will slide: A. Anteriorly B. Superiorly C. Inferiorly D. Posteriorly

D. Posteriorly

A 13-year-old girl has been diagnosed with structural idiopathic scoliosis. The physical therapist notices a left thoracolumbar structural scoliosis. Which postural deviation would be expected in this patient: A. The body of the thoracic vertebrae rotated to the right B. High right shoulder C. Spinous processes of thoracic spine rotated to the left D. Posteriorly protruding left scapulae

D. Posteriorly protruding left scapulae

After gait training a patient with a new below knee prosthesis, you notice redness along the patellar tendon and medial tibial flare. This would indicate A. The socket is too small and the residual limb is not seated properly B. The socket is too large and pistoning is occurring C. Improper weight distribution during stance D. Pressure tolerant weight bearing during stance

D. Pressure tolerant weight bearing during stance

A physical therapist examines the viscosity and color of a sputum sample after completing postural drainage activities. The sputum is yellowish-greenish color and is very thick. The PT can best describe the sputum as: A. Fetid B. Frothy C. Mucoid D. Purulent

D. Purulent

A PT is using a new special screening test to help determine if a tennis player has a rotator cuff tear among other possibilities. The test has a very high sensitivity of 0.93 (or93%). If the results of the test are negative, the PT should: A. Try another special test for rotator cuff tear B. Consider the test results inconclusive C. Rule in rotator cuff tear D. Rule out rotator cuff tear

D. Rule out rotator cuff tear

A 68-year-old male patient comes to the clinic with a history of cognitive deficits and Alzheimer disease. The patient's chief complaint is urinary incontinence. Which of the following is MOST appropriate intervention for this patient? A. Use of absorbent pads B. Pelvic floor exercises targeting levator ani muscles C.Breathing exercises to develop sphincter control D. Scheduled toileting and prompted voiding

D. Scheduled toileting and prompted voiding

During an evaluation, the PT checks the active and passive ankle ROM. The patient lacks 10 degrees of passive ankle dorsiflexion. The same limitation of 10 degrees in range of motion limitation is present whether the knee is extended or flexed. The muscle MOST likely contributing to the restriction in passive range of motion is? A. Tibialis anterior B. Hamstrings C. Gastrocnemius D. Soleus

D. Soleus

A 25-year-old male soccer player had to undergo an ACL reconstruction surgery. It has been 3 weeks since the surgery and he is undergoing PT treatment. Which of the following is LEAST likely to be contraindicated while exercising this patient? A. Short arc quads between 30 degree to full extension B. Closed chain strengthening exercises of quadriceps between 60 to 90 degree of knee flexion C. 5 lbs weights at ankle during quadriceps strengthening D. Squatting with knees staying posterior to the toes

D. Squatting with knees staying posterior to the toes

A 60-year-old male suffered a left sided CVA that caused weakness of the right-side of the body. His ankle brachial index is measured to be 0.94. The PT assesses his wound on the lower back as shown in the picture. Which of the following is MOST appropriate regarding the ulcer? A. Unstageable pressure ulcer B. Stage I pressure ulcer C. Stage III pressure ulcer D. Stage II pressure ulcer

D. Stage II pressure ulcer

A female patient is referred to her local clinic with lymphedema. The PT notices she has notable swelling that is hard and fibrotic, has a positive Stemmer sign, and hyperkeratosis of skin. Which of the following stages of lymphedema would the patient MOST likely be in? A. Stage 0 B. Stage I C. Stage II D. Stage III

D. Stage III

A patient presents with dizziness with rolling in bed and bending forward to load the dishwasher. On assessment, the patient tests positive for the dix hall pike test on the right side. Which of the following is MOST likely expected to be present in this patient? A.Pendulum nystagmus B.No nystagmus C. Down beating torsional nystagmus D. Up beating torsional nystagmus

D. Up beating torsional nystagmus

A patient presents to the PT clinic with chief concerns of loss of balance. He reports his symptoms have been gradual in onset but have progressively worsened. The PT begins the assessments and finds that the patient also presents with deficits of the left facial nerve. Which of the following is a potential cause of patient's presentation? A. Meniere's disease B. Vestibular neuritis C. Right-sided CVA D. Vestibular Schwannoma

D. Vestibular Schwannoma

A researcher is collecting data on 100 patients with venous insufficiency. Out of the 100 patients with venous insufficiency, 70 patients had a positive doppler test, and 30 had a negative doppler test. What is the sensitivity of the test? A.20% B.80% C.40% D.70%

D.70%

A patient with low back pain patient scored 18 points on the OswestryDisability Questionnaire. 4 weeks later, the score is 50 points Which of the following is the BEST option for the physical therapist? A.Continue physical therapy until the patient returns to a score of 60 B.Discharge the patient as there is an improvement C.Document the improvement and start home exercise program D.Ask the patient to have a physician consult

D.Ask the patient to have a physician consult

An elderly female patient reports to a PTA that her children are neglectful and sometimes abusive when she requires help at home. The patient rolls up her sleeve to reveal bruising. Which is the most appropriate action for the PTA to take FIRST? A.Alert the PT of the situation and allow the PT to decide the next step B.Wait until witnessing interactions of the patient and her children in order to establish reasonable suspicion C.Contact the patient's family and discuss her statements in a private conversation D.Contact an elderly abuse hotline or report to the appropriate authority

D.Contact an elderly abuse hotline or report to the appropriate authority

A 20-year-old patient has been diagnosed with a C8 complete lesion after sustaining a fall from a ladder. Presence of the which of the following is MOST likely expected to be seen in this case to confirm the diagnoses? A.Absent triceps reflex B.Absent sensation on the deltoid region C.Weakness of the elbow flexors D.Inability to extend knee

D.Inability to extend knee

A 58-year-old male patient is 3 weeks post right TKA. The PT would want to continue addressing intervention techniques in effort to return the patient to prior level of function and eliminate any discomfort. Which of the following would NOT be appropriate to include in the POC? A. Quad/hamstring sets B.Ankle pumps C.Pain modulation D.Proprioceptive training

D.Proprioceptive training

A 52-year male has a history of COPD with episodic expiratory wheezing, tightness in the chest, and coughing that is worse at night. During exacerbations, the patient experiences dyspnea at rest and uses accessory muscles of respiration. Which of the following changes are MOST LIKELY on pulmonary function tests during exacerbation of symptoms? A.Increased FEV1 and reduced RV B.Reduced FEV1 and reduced RV C.Increased FEV1 and increased RV D.Reduced FEV1 and increased RV

D.Reduced FEV1 and increased RV

A 32-year-old female patient presents with the presence of abnormal reflexes. She also notes "weird" sensations throughout her body. Her lab findings indicate increased IgG in the cerebrospinal fluid. During the assessment, she asks the PT to decrease the temperature of the treatment room as she was heat intolerant. Considering the patient's presentation so far, which signs and symptoms are LEAST likely expected to be seen in this case? A.Ataxia B.Optic neuritis C.Trigeminal neuritis D.Rigidity

D.Rigidity

A PT is performing an exercise stress test on a 45-year-old male patient. The patient resting vitals are BP 130/90 mm Hg. HR 75 bpm and RR 24 breaths/min. Which of the following is an ABNORMAL response to vigorous aerobic exercise? A.Diastolic blood decreases to 88 mm Hg B.Diastolic blood pressure increases to 100 mm Hg C.Respiratory rate increases to 34 breaths/minute D.Systolic blood pressure decreases to 100 mm Hg

D.Systolic blood pressure decreases to 100 mm Hg

A PT is evaluating a 34-year-old female patient with a vague diagnosis of low back pain. The patient displays a positive Thomas test. Which sub phase of the gait cycle will MOST likely show limitation in the hip ROM? A.Loading response B.Initial contact C.Midstance D.Terminal stance

D.Terminal stance

A PT is evaluating a 34-year-old female patient with a vague diagnosis of low back pain. The patient displays a positive Thomas test. Which sub phase of the gait cycle will MOST likely show limitation in the hip range of motion? A.Loading response B.Initial contact C.Mid stance D.Terminal stance

D.Terminal stance

Out of the following research questions, which one of them represents a quantitative research design? A. What are the perceptions of infant lower limb amputations in the United States? B. What are the experiences of patients with limb loss in returning to sports? C. What are the opinions of physical therapists treating lower limb amputation patients in an acute care setting? D.What is the relationship between level of lower limb amputation and walking distance in lower limb amputations?

D.What is the relationship between level of lower limb amputation and walking distance in lower limb amputations?

differentiate between deductive and inductive reasoning

Deductive: analyzes problems in depth and applies info and theoretical models to practical situations; conclusions are drawn based on information, theoretical models, laws, rules, or accepted principles Inductive: observes similarities and can develop theoretical models to explain relationships; conclusions are drawn by extrapolating specific situations to larger circumstances, does not guarantee the conclusion

When exercising a pt in an aquatic environment, what are the factors that can make the activity easier to perform? More difficult to perform?

Easier: movement horizontal to or upward toward the water surface, use of a flotation device, a paddle turned to slice through water, decreased speed of movement, exercising in deeper water Harder: movement downward to surface of water, increased speed of movement, use of fins, paddles, and boots to increase resistance, shallower water

For an anteriorly displaced articular disc at the TMJ, what is the primary joint mobilization technique?

Primary glide is inferior, which gaps the joint, stretches the capsule, and allows relocation of the disc

Neuro Case 1: 63 year old male. Pt referred to IP PT secondary to R ischemic stroke with L hemiparesis, admitted 3 days ago with intense HA and LUE>LE weakness exhibits L neglect. PMHx HTN, B knee OA, R knee arthroscopy. He is a college basketball coach, married and lives in a 2-story home. The patient is making progress with midline orientation in sitting, but once you transition them from sit to stand you notice they forcefully shift weight from the R to L side. You are concerned about Pusher's syndrome. Which of the following interventions would be BEST to maintain a midline posture? a. Perturbation training with weight shifts in all directions b. Visual feedback with a mirror c. Use of a cane on the involved side d. Cueing the patient to look to the uninvolved side

b. Visual feedback with a mirror


संबंधित स्टडी सेट्स

Chapter 7 - Segmentation, Targeting, and Positioning

View Set

Business Management 1/2 The external environment

View Set

How often must you renew your license?

View Set

Maternal-Newborn Ch 26 Wellness and health promotion

View Set

ECON: Unit 7-8 (Practice Set, Practice Quiz, Quiz)

View Set

Marketing Final Practice Questions

View Set

Biology Lecture 12 Clicker Questions

View Set